You are on page 1of 259

StricUv accor1Hng to the

NEW SYLLABUS

'I

Design of
.1:>l te el St ru ctu re
. "~- ,r

{For Diplom a & Polytec hnic Studen ts)


CIVIL ENGINEERING

R.K.Sa xena
D E S IG N .
OF .,

. =~: ---~-~_____, ' f

-
---------------~------------- .
fl .

/ · Ci vi l En gi ne er in g G ro up /

1
--r--ne--w --
r-St--ric--tly--as--pe --bu--s~
,y lla 1
w .e .f. ac ad e1 ni c ye ar 20 16 -1 7 t

~---------------------~
1

R it u Pu bli c•a tio n

l . !
SYLLABUS
Cha pte r 1 : Int rod uct ion
Types of sections used , Grades of steel and strength characteristics;
advantages and
disadvantages of steel as const tuctio n material; Use of steel tao1e and
relev ant I.S.
code; Types of loads on_steel structure and its I.S. code speci ficati
on.
Cha pte r 2 : Con nec tion s
Rive ted conn ectio ns, Types of rivets and their use, Type s of rivet
ed joint and its
failure, Strength of riveted joint and efficiency of a riveted joint. Assumptio
ns in theory
of riveted joint. Desig n of riveted joint for axially loaded member. Weld
ed connection
- Intro ducti on, Perm issib le stress in weld , stren gth of weld , adva
ntage s and
disadvantages ofw~ ldedj oint. Types of welds and their symbols. Desig
n of fillet weld_
and butt weld subje cted to axial load.

Cha pte r 3 : Des ign of Ten sion Me mb er


Types of sections used, permissible stresses in axial tensi on and gross
and net cross-
sectional ~rea of tension mem ber

Analysis and Desi gn of tensi on meml?er with weld ed and rivet


ed connection.
Introduction to Lug Angle and Tens ion Splice.

Cha pter 4 : Des ign of Com ,res sion Me mb er


-
Angle struts, Types of sections used, Effective length, Radi us of gyrat ., . '
ion, Slenderness
ratio and its limit, Permissible comp ressi ve stresses.

Analysis and Design of axially loaded angle struts with weld ed and rivete
d connection.
Stanchion and Columns, types of sections used; simple and built-up sectio
ns, effec~ve
length, Anal ysis and desig n of axial ly loade d colum n, Intro ducti on
to lacing and
battening. (No numerical prob lems on Lacin g and Batte ning)

Cha pte r 5 : Ste el Ro of Tru ss -l


Types of steel roof truss and its selec tion criter ia. ' II
Calcu lation of panel point load for Dead load, Live load and Wind load as per
I.S.
87 5-198 7, Analy sis and Desig n of steel roof truss. Design of Angle purlin as per
I.S.
Arran geme nt of memb ers at supports.

Cha pter 6 : Bea ms


Different steel sections used; Simple and bcilt-up sections, Permissible bending stresse
s.
Desig n of sin1ple beatns, check for shear only. Design of built-up beatns (Symm
etrical
I Secti on with cover plates only), Check for shear.only.

Introd uction to Plate Girder /: Various components and their functions. (No numer
ical
probl ems on Plate Girde r)

Cha pter 7 : Col umn Bas es


Type s of colum n bases , desig n of slab base and concrete block, introduction
to
gusse ted base. (No nume rical problems on Gusseted Base)

□□□
I

- -.~
-~ - ...
~
~ --._--.-;
CONTENTS
1. 1- 11
Ch ap te r 1 : In tr.o du ct io n
1-1
1.1 Introduction
1-3
1.2 Types :of Sections .Us ed
ristics 1-5
1.3 Grades of Steel and Strength Characte
1-7
vru:itages and
_ Dis adv ant age s of Ste el as a Co nst ruc tio n Ma ter ial
1.4 · Ad
1-7
LS. Co des
1.5 Use of Steel Table and Relevant
ures 1-8
1.6 Types of Lo ads on Steel Struct
2. 1- 40
Ch ap te r 2 : C on.ne ct io ns
2-1 -
2.1 . Introduction
2.2 Rivets 2-3

2.3 Types of R!v ete d Joi nt 2-5

2.4 . Definitions 2-6


and Ed ge · Dis tan ces 2-6
2.5 LS. Clauses for Rivet Diameter, Pit ch
2.6 Failure of Riveted Joints 2-8

2. 7 Strength of a Riveted Joi nt 2-8


ets
-2.8 Maximum Permissible Stresses in Riv 2-9
ess in Co nne cte d Par t
2.9 · Maximum Permissible Be ari ng Str 2-9
2.10 Tacking Rivets 2-9
2.11 Eff icie ncy of a Riveted Joi nt . 2-10
Joi nts
2.12 Assumptio ns in Design of Riv ete d 2-10
y Lo ade d Me mb ers
2.13 Design of Riveted Joints for Ax iall 2-14
2.14 De fin itio n of Welded Joi nt · ---2-21
ns
2.15 Advantages of Welded Co nne ctio 2-21
ns
2.16 Disadvantages of Welded Co nne ctio 2-22
-
. __2·17 _!'ypes of Welded -€ onrre-crio
ns 2-22
2.I8 Symbols Us ed for Welds •
2-23
2.19 Permissible stress and strength of fillet weld 2-24
2.20 Design of welded joints 2-25
Chapte r 3 : Design of Tension Membe r 3.1-34
3. 1 Definition ·f ,
3-1
3.2 Types of Sections Used 3-2
3.3 Strength of Tension Members 3-3
3.4 Permissible Stress in Axial Tension 3-5
3.5 Net Sectional Area 3-5
3.6 Design of Axially Loaded Tension Member 3-15

3.7 Lug Angle 3-29

3.8 Tension Sp1ices 3-30

Chapter 4 : Design of Compression Member 4.1-51

4.1 Definition 4-2

4.2 Types of Sections 4-3

4.3 Effective Length 4-3

Radius of Gyration 4:11


4.4
Slenderness Ratio 0,..) 4-11
4.5
4-12
4.6 Angle Struts
4-13
4.7 Maximum Slenderness Ratio
- ~4-14- ·
4.8 · Increase of Stresses
4-14
4.9 Strength of Axially Loaded Compression Member
4-24
4.1 O Design of Axially Loaded Compression Members
4.11 Lacing and Battening 4-4-1 I
5.1-67
Chapte r 5 .;_ Steel Roof Truss
- -~ -- s:-i- -
5.1 Introduction
5-4
5.2 Component Parts of a Steel Roof Truss
5-5
5.3 Different Types of Trusses
,--

5.4 Selection Criteria of the Type of Truss -


5-6
5.5 Terms used in Roof Trusses
5-6
5.6 Loads on Roof Truss
5-8
5.7 Wind Loads as per IS : 875 - 1987
5-10
5.8 Design Wind Speed (V 2 )
5-12
5.9 Procedure for Calculation of Wind Load
5-15
5.10 Procedure of Designin g A Steel Roof Truss
5-21
5.11 Design of Angle Purlins
5-27
Chapt er 6 : Beam s
6.1-28
6.1 Introduction
6-1
6.2 Types of Beams
6-3
6.3 Different Steel Sections Used As A Beam ·
6-3
6.4 Permissible Bending Stress
6-4
6.5 Effective Span
6-4
6.6 Maximum and Allowable Deflection
6-4
6.7 . Design of Laterally Restrained Beams
6-5.
6.8 Compound or Built-up Beams
6-10
6.9 Procedure for the Design of a Built-Up Section 6-11
6.10 Plate Girders
6-21
Chapt er 7 : Colum n Bases 7.1-~1
7.1 Introduction
7-1
7.2 Types of Column Bases
·7-2
7.3 Slab Base
7-2
7.4 · Design of A Slab Base and Concrete Block 7-2
7.5 Gusseted Base
7-9
7.6 Design of a Gusseted \ase and Concrete Block 7-9

□□□
Chapter

[ Introduction
1.1 Introduction

• A steel structure is an assemblage of a group of members (elements) expected to sustain


their share of applied forces and to transfer them safely to the ground. Depending on
the orientation of the member in the structure and its structural use, the member is
subjected to forces, either axial, bending, or torsion, or a combina-tion thereof. Axial
load can be either tensile or compressive, and accordingly the members are called tension
members or compression members,l An example of a tension member is a tie and that
of a compression member is a strJt. These are used in trusses. Another example of a
compression member is a column or post used in buildings. Primarily, all the steel structures
are constructed with elements sucp as tension members-members subjected to tensile
I ,

forces; compression mem-bers-~embers subjected to compressive forces; or flexural


I

members-members subjected to bending. The elements of a steel structure, as discussed


above, are rolled to a basic cross section in a mill, and worked to the desired size and
form in a fabricating shop or site. These elements are connected by using rivets, bolts,
pins or welds to form the structure, and the connections so formed are called joints.
Depending upon the fixity provided, the connections are classified as rigid-can transfer
moments; flexible-can transfer axial loads (shears); or semi-rigid-that fall in between
rigid and flexible.
• A significant difference between steel and concrete constructions is that the designer
has more control over the shape of reinforced cement concrete elements. For building
a steel structure, the designer is normally compelJed to use standard rolled sections.
Fortunately, the variety of steel sections available is so great that any desired structural
effect can be achieved in steel. The main adyantages of steel structures are their smaller
weight-to-strength ratio, speed of erection and dismantling~ and its scrap value. However,
the faster degradation of their strength in the events of fire, requirement of skilled,
personnel and the accuracy desired in fabrication are the major drawbacks.
-
Design of Steel Stru ctur es [ 1-2 ]
Int rod uct ion
• Steel, as a building, material, has
been used extensively in var iou s typ
es of structures.
som e of the examples of civil engine
erin
g works in steel are high-rise buildin
g skeletons,
industrial buildings, transmission tow
ers, railway ori dge s, ove rhe ad tan
ks, chimneys
(stacks), bunkers and silos. Figure
1.1 illustrates some of the typical stee
l structures.
Steel structures can be divided into
two principal groups :
1. Steel structures, wh ich are made larg
ely of plates or sheets, such as tanks,
bins,
chimneys and ste el roo f covering of
large buildings, and
2. Fra me d structures, which are charac
terized by assemblies of tension, com
-pression
and flexural members such as, truss-fr
ame, rigid frames, girders and colum
ns, etc.

---= ~.......- Ridg e stru t

Eav e stru t
Roo f truss
Kne e brac e

Lon gitu dina l


ben t
Side brac ing
(a) Fram ed bui! dlng
(b) Indu stria l buil ding

Vertical Siiffaner.

Plat e gird er Circ ular


gird er
Hor izon tal
stiffener Brac ing
Colu mn
Floo r bea m
(c) Rail-road brid ge
(d) Ove rhea d wat er tank

Cyli ndri cal sha ft

Bree ch ope ning Tower leg


Flared port ion .,.,......,-4-_ Bracing
4 - -_ -~_
_ , _ _ , __
_,.-
_...::,.
Clea n-ou t doo r

(e) Self -sup port ing steei stac k


(f) Tran smis sion line tow er

Exa mpl es of stee l stru ctur es

j
Design of Steel Structures [ 1-3 ] Introduction

• The design of steel structures involves the planning of the structure for specific ~ur-
poses, proportioning of members to carry loads in the most economical man-ner, and
considerations for erection at site. First, the stmcture should serve the purpose for which
it is intended and this is achieved by proper functional plan-ning. Secondly, it should
have adequate strength to withstand direct and induced forces to which it may be
subjected during its lifespan. An inadequate assessment of forces and their effects on
the structure may lead to excessive deformation and its failure. Therefore, the design
of structures includes functional planning, acknowledgement of the various forces, strength
of materials and the design methods. In addition, the structure should be economical and
easy to erect. An economical structure requires an efficient use of steel and skilled and
unskilled labour. Although this objective can usually be accomplished by a design that
requires a minimum amount of steel, saving can often be realized by using more steel
if it results in a simpler structural form with less fabrication. In fact, as of today, materials
account for one-third or less of the cost of a typical steel struc-ture, whereas labour
costs can account for 60 per cent or more. This implies that economy in steel work is
realized when fabrication is minimized. •
• Structural steel has been used in the construction of structures for well over a century.
• It is perhaps the most versatile of structural materials and has been used extensively
in the construction of multi-storeyed buildings, railways, bridges, industrial structures,
transmission towers, overhead tanks, chimneys, bunkers, silos etc.

1.2 Types of Sections Used


• The following types of sections are standardized by the Indian Standard s
Institution.
1. I-Sections :
(a) Indian Standard Junior Beams (ISJB).

(b) Indian Standard Light Beams (ISLB).

(c) Indian Standard Medium Weight Beams (ISMB);

(d) Indian Standard Wide Flange Beams (ISWB).

(e) Indian Standard Column Sections (SC) or (ISSC).

(f) . Indian Standard Heavy Weight Beams (ISHB).


Des ign of Stee l Stru ctures 1-4
Introduction
1+ -b= Wi dth -.t

Top flange .
Fi lle t--
1
Webc.·=:=:::::..==~h =1Deplh
Bottom
flange

Fig. 1.1 : Bea m Section


• All above I-se ctio ns are designated
with the depth of the. res pec tive sec
• tion in mm.
Fo r example, ISL B 200 wh ere 200 ind
icates the depth in mil ime tres . Th ese
sections are
also kno wn as Ro lled Steel Joists
(R.S.J.)~
2. Ch ann el Sec tio ns
(a) Ind ian Sta nda rd Jun ior Chann
el (ISJC).
(b) Indian Sta nda rd Lig ht Channel
(ISLC).
( c) Ind ian Sta nda rd Me diu m We
ight Ch ann el (ISMC).
(d) Ind ian Sta nda rd Me diu m Weigh
t Ch ann el with par alle l flange (M CP
).
b =Width

1~ ~n ge

· h ::. Oeptii · Web

·1.____. .
_ Fillet

Fig. 1.2 : Ch ann el Sec tion


3. An gle Sec tio ns
(a) Indian Standard Equal Angles.
(b) Indian Standard Unequal Angle
s.
• Angle sections are designated by abb
rev iation ISA along wit h the lengths of
both legs
and their thickness.
• Fo r example, ISA 50 x SO >< 6 mm
means an equal-angle section of 6 mm
thickness
and with both legs 50 mm long.
-
In tro du cti on
Desig n of Steel Structures [ 1-5 ]

f
f
I
t

l - - - ~+
,. b
b = Width of shorter leg
I = Length of longer ieg io n{ / - b) ·
(/ = b) {b ) Unequal Angle Se ct
(a ) equal Angle Section
Fig. 1.3

4. Tee Se ct io ns
e bars - (ISNT).
(a) Indian Standard Normal Te
ge Tee bars - (ISHT).
(b) Indian Standard Wide Flan
ed Tee bars - (ISST).
(c) Indian Standard Lo ng Legg
bars - (ISLT).
(d) Indian Standard Light Tee
e bars - (ISJT).
( e) Indian Standard Junior Te
ed Tee bars - (ISDT).
(t) Indian Standard Deep Legg
Weight Tee bars - (ISMT).
(g) Indian Standard Slit Medium
).
rs from I sections - (ISHT
(h) Indian Standard Slit Tee ba
,.. __ b
... _b =W id th +t
T ·---
F il/ et -+ -- - Flange· I
h
h = Depth

l __
Web

L ep Legged Tee Bar


r ( b) Slit Tee Bar and De
(a ) Rolled Norm..al Tee Ba
Fig. 1.4

St ee l an d St re ng th C ha ra c.tcristics
1.3 G ra de s of
gths.
the va rio us typ es of str uctural steels and their stren
Table 1.1 gives
-Des1g
- -- Tab Ie 1•1 : Types of Structural Steel
~

Nominal Tensile Y eild stress


Class of product
of steel thickness strength (N/mm 2) ·
Type
. (mm) (N/mm 2 )

18 :22s11s Plates, sections. Up to 20 410 to 530 250


(Standard > 20 to 40 410 to 530 240
angles, tees, beams.
quality) 410 to 530 230
channels, etc. flats. Over 40
Bars - round.
Up to 20 410 to 530 250
square and
Over 20 410 to 530 240
hexagonal
IS: 961/75
Plates, sections. Up to 28 570 350
(High tensile)
St 58 HT angles, beams, > 28 to 45 570 340
channels etc. > 45 to 63 570 320
bars, flats. Over 63 540 290
St55 HTW Plates, sections, Up to 16 540 350
bars, flats > 16 to 32 540 340
> 32 to 63 510 330
Over 63 490 280
IS : 2062/84
(Fusion welding Plates, sections - Up to 20 410 250
quality) angles, beams, tees, > 20 to 40 410 240
etc. flats Over 40 410 230
IS: 1977/75
(Ordinary quality) Plates, sections - · Up to 20 410 - 530 250
Fe _410-0 angles, beams etc. · . > 20 to 40 410 - 530 240
Over 40 410 - 530 230
Flats, bars Up to 20 410 - 530 250
Over 20 410 - 530 240
IS: 8500/77
Fe440 HTI Plates, sections - <6
- 440 to 560 300
and angles, beams. > '6 to 20 440 .to 560 300
Fe 440 HT2 channels etc-. > 20 to 40 440 to 560 290
bars; flats > 40 to 63 I
440 to 560 280
Fe 540 HT, Plates, sections -
I

<6 540 to 660


Fe 540 HTA . 410
angles, beams, > 6 to 20
and 540 to .650 400
channels, etc. > 20 to 40
Fe 540 HTB 540 to 660 39.0
bars, _
flats > 40 to 63
Fe 570 HT 540 to 660 380
-do-
<6 570 to 720 450
> 6 to 20 570 to 720 440
> 20 to 40 570 to 720 430
> 40 to 63
Fe 590 HT . -do- <6
570 to 720 420
590 to 740 490
> 6 to 20
Fe 640 HT -do- 590 to 740 480
. <6 .
640 to 790 540
> 6 to 20 640 to 790 530
Design of Steel Structures
[ 1-7 ] Introduction

- t.4 Advantages and Disadvantages of Steel as a Construction Material


.Advantages :
1. Extensively useful for large span industrial stru t
c ures, bn·.dges, towers an d
communication networks, steel overhead tanks.

Steel has many good mechanica l properties like malleabi'l'ty d t·i· st


· 1 , UC 1 1ty, e1a o-p Iast'lCl·ty
(i.e. more ultimate strength and too large strains).
3. It is most appropriate material to construct e rth k .
a qua e resistant structures due· to
more ductile nature.
4. It is easy to fabricate by riveting or welding to any desired shape.
5.Can sustain tension, compression, shear bending and torsional forces.
Disadvantages :
1. Steel is a very costly material.

2. It is susceptible to corrosion and hence requires corrosion treatment periodically.


3. It requires skill labour for erection.
4. Creates noise and requires electricity during connection of members.

1.5 Use of Steel Table and Relevant I.S. Codes

• Steel table gives the general properties of the standard steel sections.
• The properties are designation, weight per metre, sectional area, depth of section, width
and thickness of flange, thickness of web, moments of inertia, radii of gyration, moduli
of section etc.
• Referring to steel tables one can easily choose the section according to the design
requirements.
• The properti-: :; of some of the sections are given at the end of this book in Appendix
C for ready reference.

• Bureau of Indian Standards (BIS) has evolved a rational, efficient and economical series
of Indian Standards.
1. For Steel :
(A) Structural Steel as per
IS : 226, IS : 2062, IS : 3502, IS : 1977, IS : 961, IS : 8500.
~~~~~~
!?_esign of Stee l Strn ctures~ - - ~ - --..1.[.:...l-...::.8..J.]_ _ _ _ _ _ _ _ _In_tr..;.,od:.:uct10
~
(B) Stee l for reinforced con crete
IS : 432, IS : 1139, IS : 1786, IS : 2090.
(C) Steel for bars, rivets etc.
IS: 1148, 1149, 1570, 2073, 7383, 443 l, 4432
and 5517.
(D) Steel for tubes and pipes
IS: 1239, 1914, 1978.
2. For Code of Practice for Design of Stiel Str
uctures :
IS : 800 - 1984.
3. For size of weld and stresses in weld :
IS : 816 - 1969.
4. For code of practice for design loads :
IS : 875 - 1987.
Part l : Dead loads - Unit weights of building
materials an~ stored materials.
Part 2 : Imposed loads.
Part 3 : Wind loads.
Part 4 : Snow loads.
Part 5 : Special loads and load combinations.

1.6 Types of Loads on Steel Structures

The various loacjs which are likely to act on


the steel st~cture are as given below :
1. I;)ead load.
2. Live load (Imposed load).
3. Wind load .
4. Snow load.
5. Seismic load.
1. Dead Load :
• The dead load s (viz. the'self weight of the stru
ctural members), superimposed dead
load s and loads due to filling materials are refe
rred as permanent loads. '
• Dead load s are loads which are constant in mag
nitude and fixed in position throughout
the lifetime of the structure.
Dead loads in a building comprise the self
weight of the structure and all 0th er
-
pcsign of Steel Structures [ 1-9] Introduction
superimposed dead loads (viz. all permanent constructions and installations including
weight of walls, partitions, floors and roofs).

• IS : 875 (Part I) - 1987 gives the unit weight of building materials, building parts and
components.
2, Live Load :

• Live loads are the loads which vary in magnitude and/or in positions .

• Live loads are also known as imposed or transient loads .

• Live loads include any external loads imposed upon the structure when it is serving its
normal purpose.

• Live loads are assumed to be produced by intended use of occupancy in buildings including
distributed, concentrated, impact and vibration and snow loads.
• Live loads are expressed as uniformly distributed static loads (u.d./.).
• Live loads include the weight of materials stored, furniture and movable equipments.
• Code IS : 875 (PART B) - 1987 defines the principal occupancy for which a building
or part of a building is used or intended to be used. The buildings are classified according
to occupancy as under as per IS : 875 (part 2) 1987.
1. Assembly buildings.
2. Business buildings.
3. Educational buildings.
4. Industrial buildings.
5. Institutional buildings.
6; Mercantile buildings.
7. Residential buildings.
8. Storage buildings.
• Imposed floor loads for different occupancies are given in IS : 875 (part 2) -1987.
3. Wind load :
• The wind exerts pressure on the structures.
• The pressure exerted may be both external and internal.
• The external pressure will depend upon the geographical location of the structure, Its
height and its proximity to other structures which cause a hindrance to the air flow.
• The internal wind pressure will depend upon the permeability of the structure.
- D . n of Steel Structures
es1g . .
[ 1-1 0 l
lntrod~~
·th large openings m its cla dd' swill be subjected to more intemal _
mg . . pres · ·
A structure w1
• .
ft \1e structures, the com b.med effect of external and mternal Pres
. s~.
• In the des,gn
. d ·11
° . . .
er reco m m enda
· . · . ~Ute
of wm w1 be taken . This is done as p tions of_IS : 87 _5 ~
19
i1
(Part 3).

4. Snow loads :
__ , lo
• In the areas of snow fall ' an a\\( ,wance for •~now ads- will be taken as pet IS-\·%
· . .
. ·.· . . .7s
· d ·t
. , .
Actual loads caused by snow h
• d,:p.ends upon t e sh ape of
.
th e .ro of an 1 s. ~apac1ty-:. \-'
retain the snow. . · · •
. . -~
It is a common practic .- · b. in the wind loads with . h · · ·-
• e of not com m g t e snow ·loads.
.
• However, when the· y are assu d · t vertically over an entire ar . .
me to ac _ ea, due-allowanc
for this combined effect shall . . e
be made. .
5. Seismic loads :

• Structures s1. tuated m


. th.
e reg1·ons sub,1 ected to earthquakes shall be
J -designed to resist
the hon·zontal .c1orce pro duced 1·n the structures du ..

e to the earthquake tremor


These loads shall be assumed . . s.
• as per IS : l 893-200 2 . (Par
t _I) "Criteria for Earthquake
Resistant Design of Structures
_ Part 1 : Genera\ Provision
s 'and.Buildings". Reference
may be made in this regard
to IS : 875_for full details.

Important Points
• Types of Sections used in ste
el structures are \-sections, Ch
annel sections, Angle sections
and Tee sections.
• Steel is useful for large span
industrial structures, bridge
s, communication towers ana
overhead tanks .
•~ Steel table gives the genera\ pr
operties of like weight per m
etre, sectional area, dimension
moment of inertia, modulus s,
of section and radii of gyra
tion etc.
• IS : 800 -1984 is the code
of practice for Design of
Steel structures.
• Types of loads on steel struc
tures are : Dead load, Live
Seismic load. load, Wind load, Snow \oad
and

Exercise Problems

l. What •are the types of load


s to be considered while de
2. signing the structures ?
State four type.s of loads to
which structures are subjec
ted.
Design of Steel Structures (. l- l l } Introduction
3. State any two/four types of structural steel sections.
4. Draw .neat sketch of unequal angle section ·showing a\\ components and giving
geometrical properties.
5. Draw a neat sketch of LS, channel section, l ·section, T section.
6. ·oraw neat sketches of rolled steel channel sections an~. eq\la\ ang\e section showing
•their all components and giving geometrical properties~
7. State LS. code useful for getting load on th~-.s tiutitures.
. 8. State any one advantage and any one disadvantage ~f r1se of steel as construction
material.

9. Enumerate any four . types of forces to be considered in designing steel structures.


10. State the full form of lSWB and ISMC.
11. State any four .rolled · steel sections used and draw the sketch of any one.
12. Enlist types of loads to be considered while designing steel structures.
13. State the meaning of ISLB and ISHB.
14. State any two grades of steel and characteristic. stre~·gth in stee\ structures~
15. What are the loads on steel structures and its l.S. Code provision'? (Any two)
16. Draw neat labelled sketch of I-section of steel.
17. .·State any two advantages of steel as construction material.
18. State use of I.S. 87 S.

□□□
-
Chapter
Connections
(A ) R IV E T E D C O N
N E C T IO N S
1. 1 Introduction
• Although riv et ed cons
truction is obsolete, bu
t an understa nding of
de si gn is es se nt ia l for its behaviour and
the strength evaluation
and rehabilitation of ol
fo r the sake of m ak der structures. Just
in g th e reader familiar
with riveted connectio
description about rivet n design a brief
s and their patterns are
presented. Since the an
of riveted co nn ec tio ns alysis and design
is same as that for ordi
nary bolts, the design
do ne sim ilar to ordina and details may be
ry bolts.
• A rivet is made up of
a round ductile steel ba
r piece (mild or high te
w ith a he ad at on e en nsile) called shank,
d. Th e head can be of
different shapes as show
( d) . Th e us ua l form of n in Figs. 5.1 (a)-
rivet head employed in
structural steel constru
he ad . Th e snap head ction is the snap
s and pan heads form
a projec-ti on beyond
w he re this is an ob je the plate face, and
ct io n- as in bearings,
where continuity betw
or betw ee n plate and een plate and plate,
masonry, is ne ce ss ar
y- a countersunk head
is employed.

(a) Snap
(b) Pan (c) Flat countersunk

\•Grip•\

Length
Manufactured
head
tp
- Shank
Initial clearance
tp-Nominal diameter of
rivet
(d) Round countersu d-Gross diameter of riv
nk et
(e)
Fig. : Rivet types, gr ip
an d length
[ 2-2 Con
] ._ _ _ _ _ _ _~~:: nec
:.:.:. tion
:::.:: s
.::.
of Stee l Stru ctur es
.::-= ..__ _ _ _ _.l....:'....::...l....
Desi
-~--gn
---. :.... ::..: .::.:
the parts to be connected and with
• The shank is made of the length to extend through
made at the other end.
sufficient extra length for a second head to be
market is the grip of the rivet plus the
• . The length of the rivet to be ordered from the
. The grip of the rivet is the distance
extra length required to form the second head
the shank length mst · 'de th e connect'10n
between the undersides of the two heads ' i·e ·,
long; the rivet is subjected to bending
(Fig. (e)). When the grip of the rivet becomes
grip length should not be more than
in addition to bearing and shearing stresses. The
should be proportionate ly larger. The
4 diameters. If the grip is more, then the diameter
fixing of the rivet diameter. The head
grip has , therefore, an important bearing upon the
ing (when the rivet bar is red hot) ,
at the other end can be ·formed by hand hammer
sure driving.
hydraulic pressure driving or by pneumatic pres
diameter. A hole slightly greater than
• The diameter of the shank is called the nominal
nom inal diam eter is drilled in the parts to be connected. The rivet is inserted and
the·
e process is called riveting. The rivets
the head is fom1ed at the other end. This complet
ed to a red hot state. When the rivets
may be placed in a cold state or may be heat
en field rivets or hot driven shop rivets,
are heated before driving they are called hot driv
or in the workshop. The diameter of the
depending upon if they are placed in the field
hole and is called gross diam eter. The
rivets when hot is equal to the diameter of the
rivet hole completely in the process of
hot rivet becomes plastic, expands and fills the
rivet shrinks both in length and diameter.
forming a head at the other end. On cooling, the
the connected parts become tighter,
Due to the short-ing of the rivet shank length,
an unpredictable amount in the shank
consequently resulting in the residual tension of
ted. The compression causes friction
and some compression in the plates to be connec
action. Due to the reduction in diameter
to slide between the plates and is called clamping
between the sides of the hole and the
of the slrnnk on cooling some space remains
shank.
room temperature) is limited as high
• The use of cold driven rivets (rivets driven at
temperature and it may not be feasible
pressures are required to from the head at room
the equ ipm ents for mak ing th o head in the field. The strength of the cold driven
to use
clamping force is however less (as the
rivets is more than the hot driven rivets. Their
ts. The rivet heads for small diameter
rivets do not shrink) than that of hot driven rive nd
form ed man uaIIy with an ord inar y ham mer. Such rivets are called ha
rivets can be
driven rivets.
~
Conne .t··%.,
[ 2-3 ] ·--~~~!~
h · h _, t
ted
. f Steel Structures nents w tc ar e conn ec
Design ° is made-up of. various compo ogether
t e
Eve,Y steel struc ur
• either by means of rivets, bolts, pms or by welds. ..nt
cu red by th e use of rivet s, it is called riveted. JOt
· ·nt between the members
is se
Weld ·
• If e JO t between the members is secured by the use of welds, it is ca. lled erj
th I
If the join
• ..•
joint

2.2 Rivets
nnect two or more than two steel members
in place to co
• A piece of round steel forged
together is known as nvet.
s of a 'head' an d a 'sh ank' as shown in Fig. 2.1 .
• A rivet consist
e~ of rivets :
Following are the different typ
I. Snap head.
2. Pan head.
Pan head with tapered ne
ck. ·


4. Rounded counter sunk bead
..
5· · Flat counter sunk head
6. Flat head.
Head

I
i
.. I ,

' !

·Shank
I
i
!>
I

Fig. 2.1
.
• own· . p·
All these rivets have been · sh . Ill tg. 2.2.
I . .
• Snap head rivets are the m _ost .
. com mon y used ty
pes o
f .
rivets .
.
..
• .. .
• Counter sunk head rtv ets are used where a flush s . .
. . . .ur face ts desired and flat head rivets
I
are used where the available c earance is small d 1· . . d .
. . . an im1te
• Rivets are also classified as

. (a) (i) Hot driven rivets


(ii) Cold driven rivets
(b) (i) Shop rivets (ii )
Field rivets
Connections
-
Design of Steel Structures

(c) (i) Power-driven rivets


[ 2-4]

(ii) Hand-driven rivets


}lot driven _rivets :
, These rivets are heated to red he.at before driven in position.
, This introduces a great clamping force and thereby an initial tension in the rivets is
,

caused when they are cooled down to the nonnal temperature.

T T T~ ~l 0.50
£ i
i
Cl
C:
Q)
.J
i:
~
l
1
l
T
Snap Head Pan Head Pan Head with Tapred Neck

~1.6D- .j j.-1.5 D ..j


20----, j_
T T 0.250
T
....
.r:.
0,
i:
-
J:
0,
C: =g>
Q)
~
..J
~

1 l t
Flat countersunk Flat Head

Fig. 2.2

Cold driven rivets :


• These rivets are driven in the cold state i.e. at atmospheric temperature.
• They do not introduce any clamping force to the connected parts connected and hence,
no initial tension is present in these rivets .
.• But they nee-.1 a large force to form the head of rivet. 12 to 22 mm diameter rivets can
he driven b:-: this method.
Shop rivets :
• These rivets are driven in· the workshop under better control and conditions and hence
they have greater strength.
Field rivets :

• These rivets are driven at the work site.


I
Design of Steel Structures [ 2-5 ] _ .
~
Connectio
• ntrol
• There 1s 1ess co on the process of fabrication resulting into less strength· ·
Power driven rivets :
• These rivets are driven with the hel
p of automatic machine and powe
r hammers and
hence they will have better strengt
h.
Hand driven rivets :
• These rivets are driven manually by
sledge hammers. Hence strength of
these rivets
will be less than that of power driven
rivets .
2.3 Types of Riveted Joint

• Following are the two types of rivete


d joints :
1. Lap Joint 2. Butt Joint
1. ·Lap Joint :
• In this type of joint, the plates are con
nected overlap each other. The lap joi
single row, staggered or chain rivetin nt may have
g.

I
I
I
I
I
I
I

~ -'-7=-~lt-=..-:;::..·-~--- -----,~P
-4· Overlap r-
Fig. 2.3 : Lap Joint
2. Butt Join{'-:'
',, .

• The plates to be joined together are


made to butt against each other and
the connection
is made by providing a cover plate
on one or both sides of joint.
• The butt joint may have a single row
, staggered or chain riveting.

p--w@/@/.b41,;/41N».~~~p
(a) Single Riveted Single Cover
Butt Joint ·

P-Wff@~w~~P
(b) Double Riveted Double Co
ver Bu tt Jo int
Fig. 2.4 : Butt Joint
Design of Steel Stmctures [ 2-6 ] Connections

2.4 Definitions
1. Nominal diameter:
• The diameter of the shank of a rivet before riveting is called the nominal -diameter.
• It is denoted by letter 'dn'.
2. Effective diameter or gross diameter :
• The effective diameter or gross diameter of a rivet is equal to the diameter of the hole
it fills after riveting.
• It is denoted by 'd'.

Fig. 2.5 : Gauge Line On a Joint

3. Pitch Distance :
• It is the distance between centre to centre of any two adjacent rivets and . is denoted
by 'p'. The pitch is measured in the direction of load. Hence it is also termed as
'longitudinal pitch'.
4. Gauge line :
• An axis passing through the rivets and parallel to the direction of force is called a gauge line.

5. Gauge distance :
• The normal distance between two adjacent gauge lines is called the gauge distance·.
• The gauge distance is measured in the direction transverse to load. Hence it is also
called as 'transverse pitch'.
6. Edge distance :
'
• It is the distance between the edge of a member or cover plate and the centre 0f the
nearest ho]e.

2.5 I.S. Clauses for Rivet Diameter, Pitch and Edge Distances
AS PER JS: 800 - 1984
Desi n of s1ecl Structures 2-7
1l
IS § 3.6 : Rivet diameter
d = dn + 1.5 mm,
d = dn + 2.0 mm,
where, d = Effective diameter,
and dn = Nominal diameter.
IS § 8.10.1: Pitch of rivets
(a) Min. pitch - 2.5 times the nominal dia. of
rivet
(b) Max. pitch
(i) Any two adjacent rivets-32t or 300 mm, (inc
luding tacking rivets) whichever
is less;
(ii) Rivets lying in a line parallel to the forc
e in the member,
(a) in tension - 16t or 200 mm whichever
is less,
(b) in compression - 12 .t or 200 mm
whichever is less .
where, t = Thickness of thinner outside plate. ·
IS § 8.10.2 ; Edge distance .

Dia. of rivet Dia. of hole For sheared For rolled and


or rough edge pla ne edg e
12 13.5 & below 19 17
14 15.5 25 22
16 17.5 29 25
'
18 19.5 32 29
20 21.5 32 29
22 23.5 38 32
24 25.5 ·
44 38
27 29
SI 44
30 32
57 51
•33 35
57 51

I
[ 2-8] Connections
-2.6
Design of Steel Structures
Failure of Riveted Joints
It is as shown in Fig.2.6.

(a) Tearing of plate (b) Shurtng of rivet•

(c) Bearing failure of plate


(d) Bearing fallureof rivet

(e) Edge cracking ohpllttfng failure

Fig. 2.6 : Failure of Riveted Joints

2.7 Strength of a Riveted Joint


• Consider one pitch length p of the riveted joint.
• Strength of joint per pitch smaller of following two.

I 0 1
0
Fig. 2.7
(i) The strength of plate between rivet holes in tension.
(ii) The rivet value.
(i) Strength of plate between rivet holes in tension = erat x (p-d) x t
= Smaller of following two
(ii) Rivet value
(a) Bearing strength of rivet = er pr x d x t or er p x d x t whichever is less_
(where, t is the thickness of thinner plate).
Design of Steel Structures
[ 2-9
7C 2
(b) Shearing strength of rivet = tvf x xd in single shear
4

7C 2
- 2-rvf x-x d in double shear
4

where, O'at = Allowable tensile stress in an axially loaded tension member.


cr P = Allowable bearing stress in plate.
O'pf = Allowable bearing stress in rivets.
tvf = Allowable shearing stress in rivets.

2.8 Maximum Permissible Stresses in Rivets

Type of fastener Axial tension (J'tf Shear 'tvr Bearing crpl ·

(MPa) (MPa) (MPa)


(i) Power driven
II
(a) Shop rivets 100 100 300
11

(b) Field rivets 90 90 270


'
(ii) Hand driven rivets 80 80 250

2.9 Maximum Permissible Bearing Stress in Connected Part

Type of fastener crP (MPa)

(i) Power driven rivets JOO


(ii) Hand driven rivets 250

2.10 Tacking Rivets

• Tacking rivets or stitch rivets join two individual stn1ctural members to work as 8
homogeneous .member. When the compression members are connected back to back
and it is desired that they should act as a single unit so as to prevent buckling.
• Thus tacking rivets are provided along entire length of member.
. .
1. For compress10n members where cover plates are used, the pitch shoul d no t exceed 1

32 times the thickness of outside plate or 300 mm whichever is J,e$S.

J
[ 2-1 O ]
Connections
pesign of Steel Strnctures
than 600 mm for compression memb ers.
2. Tacking rivets shall have pitch not more
n me mb ers, the pitch of tacking rivets shall not excee·d 1000 mm.
3. For tensio

2.11 Efficiency of a Riveted Jo int


ina l strengt h of a section · of a me mber is reduced due to rivet holes. The
Th e orig
• of the strength of the joint (which is we
akened
efficiency of a riveted joint is the ratio
of rivet holes) to the orig inal strength of the member without rivet holes.
by provision
at the
ve at ma xim um efficiency , the num ber of rivet holes should be minimized
To arri

weakest critical section.

ted Joints
2.12 Assumptions in De sig n of Rive
is and des ign of rive ted joi nt is very complex. However, it is simplified by
, The analys
ow :
making a few assumptions as given bel
en the
The ten sile stress is uni formly dis tributed on the portions of the plate betwe
I.
rivets.
lected.
2. The friction between the plates is neg
d on cross-section of the rivets.
3. The shear stress is uniformly distribute
4. The rivets fill the holes completely.
equally.
5. The rivets in a group share direct load
·6. Bending stress in rivets is neglected-
and the contact area is d x t where 'd' is
7. Bearing stress distributi on is uniform
the plate.
the diameter and t is the thickness of

I SOLVED EXAMPLES I
meter rivets connecting 10 mm plate and
Ex.2.1 Determine the rivet value of 18 mm dia
shear and (ii) Do uble shear. The permissible stresses for rivets in
is in (i) Single
~nd bearing are 80 MP a and 250 MP a respectively and for plate in bearing
shear
is 250 MPa.
Soln. : Gross diameter of rivet
d = dn + 1.5
= 18 + 1.5
= 19.5 mm
- ~~ ~ ~ ~ ~
"Q_esign of St ~
eel Structure-s - - . : . . - -_ _j[~2::,-1~1~
1~_ ______
Strength of rivet _
C_on_n..;.ec.;:.:t~~
:
(i) In bearing, Pb = cr
pf x d x t
= 250 x 19.S x 10
= 48750 N

(ii) ..
1t . 2
In single shear, P
8 = tvr x d
4

= 8 0•·x1t-x l9 .5 2
4

::;: 23891.8 N

(iii)
In double shear, Ps
s = 2x80x 1t xl9.5 2
4
=47783.6 N
Rivet value in sing
le shear ~ Smaller
~f (i ) and (i i)
= 23891.8 N
Rivet value in doub
le shear = Smaller
of (i) and (iii)
=47783.6 N
Ex.2.2 : Deter
mine -the lo.ad wh
ich can be transm
cover butt joint co itted per pitch le
nnected _by 22 mm ngth o f a double
thickness o f main diam eter shop rivets at
plates and cover 9 cm pitch. The
plates is 16 mm an
The allowable te d l O mm respecti
nsile strength is vely.
efficiency of the equal to l SO M
joint. Pa. Also calculate
the
Soln. : For shop rivets, th
e allowable stress
In bearing, es are
In shear, crpf ~ crP =300
MPa
'tv f = l 00 MPa
crat == 150 MPa
Strength of joint d = 22 + 1.5 = 23.S mm
:
(i)
·Strength of plate
per pitch = cr at x (p _ d) t
-
pcsigt! of Steel Structures [ 2-12]

= 150 X (90 - 23.5) X 16


Connections

= 159600 N
(ii) Strength of rivet in bearing, Pb = crp • d •t
= 300 X 23. 5 X 16
= 112800 N

Note : As the total thickness of cover plates is more than thickness of main plate, the
thinner plate will be main plate i.e:- 16 mm thick. ·

(iii) Strength
.
of rivet in double · shear, Ps
s
= 2 tvf x 7t4 xd 2

= 2xl00x 7t x23.5 2
4
= 86747 N
Strength of joint per pitch = Smaller of (i), (ii) & (iii)
= 86747 N

Strength of joint
Efficiency of joint = Streng th of pa
1 te

86747
=----
150x90x16
= 0.4 or 40%
Ex.2.3 : A double riveted double cover butt joint is used to connect plates 12 mm thick.
. If 22 mm diameter power driven shop rivets are used at 70 mm, determine the
strength of joint and its efficiency. Take crat = 150 MPa.
Sol1 • : (i) Effective dia d = dn + 1.5 = 22 + 1.5 = 23 .5 mm.
(ii) Strength of rivet in double shear (since if the rivet fails it will fail into three parts).
1! .2 7t 2
Pss =2X'tvf X -;-r.! =2xl00x-x23 .5 =86747N.
4 4.

(iii) Str@ngth of rivet in bearing Pb = cr ~f x d x t = 300 x 23.5 x 12 = 84600 N.

(Here the thickness of main plate is considered as thinner, since both the cover plates
hav~ combined thickness of 8 + 8 = 16 mm).
(iv) Rivet value, Rv = 84600 N (Minimum of (ii) and (iii) above).
I
[ 2-13 ] Connection
f Steel Structures
.
Design ° ' ~

1~ 10 ,r 10 •r 10 . ,r .,r 10 ,r 10 ~ 70 J,
1 '1

Fig. 2.8

(v) Strength of plate between the rivet holes in .tension

= 150 (70 - 23.5) X 12

= 83700 N
e.
(vi) · Strength of the joint is minimum of (iv) and (v) abov
= 83700 N

Strength of joint
(viii)
Efficiency of joint = Strength of plate

83700
-
- -x12
= -x70
150

= 0.6643 = 66.43%
Consider a single rivet line for calculation of strength.
For power driven step rivets allowable stresses
In bearing, crpf = 300 MPa
· In shearing, tvf = 100 MPa
crat = 150 MPa
Let us assume thickness of cover plate = 8 mm.
plate is not given.
Also aSsume gauge dist ance = p = 70 mm as width of
Design of Steel Structures [ 2-14 ] Connections

2.13 Design of Riveted Joints for Axially Loaded Memb ers


, The diameter of a rivet is calculated by using Unwin's formula

where, dn = Diameter of rivets in mm.

t = Thickness of plate in mm.


However, the minimum size of rivets shall be 16 mm.

. . dc . . Load
• Num ber of nvets reqmre 1or Jomt = - - - -
Rivet value

lsoLVED EXAMPLEsl
Ex. 2.4 : Find out the number of power driven field field rivets required to connect a tension
member in a roof truss to a gusset plate 6 mm thick. The tension member is a
flat of size 75 mm x 6 mm and is subjected to a pull of 48 kN.
Soln. : For power driven field rivets,
tvf = 90 MPa
crpf = 270 MPa

Diameter of rivet, dn = 6 ✓t = 6✓6 = 14.6mm (say 16 mm)

Gross diameter d = 16 + 1.5


= 17.5 mm
Strength of rivet : .
7t 2
(i) In single shear = tvf x d
4

7t 2
= 90x-xl 7.5
4

= 21647 N

(ii) In bearing = crpf x d x t


= 270 X }7.5 X 6
.=2835 0N
• n~ ·~n~~f-:_!S~t~ee:!_
D
~es1 g ctu~r~ e s ~ - - - -- - ~ [~2-~l :.:.
l ~S~tn:_1_1~
~11
S~ ]~~ ~~ ~= ~-- -... :::C ~o nn ect·
Rivet value = Smaller of (i) and (ii) ~C llls
~
= 21647 N

Nu mb er of rivets required 480 0


= - - = 2.21 (say 3) \
21647

Min. pitch = 2.5 dn


\
= 16 x 2.5 = 40 mm, say 50 mm
Edge distance = 29 mm say 40 mm

40

I .
I

Flat 75 mm x6 mm
Rivets 16 mm dia.
plate 6 mm thlc~

Fig. 2.9
Ex. 2.5 : Fig . 2.10 sho ws a joi nt in the lower
chord of .a roo f truss. De sig n the rive
ted
con nec tion if the permissible stresses
are
crat = 150 MP a
crpf = 250 MP a
1:vf = 80 MP a
B
56KN
E
E
a:,
X C
~ 112 KN
X

~
...I
284 KN
A ~ -- -- -- -~~~ -- -- -- -D O
..L 100 x 100 x 1O mm
187 KN

Fig. 2.10
Soln. : Us ing a 12 mm thic k gusset pla
te,

No min al diameter of rivet = dn = 6 ✓10


= 18.9 (say 20 mm)
-
Design of Steel Structures [ 2-16 ]
Gross diameter, d .= 20 + 1.5
Connections

= 21.S mm
Member OB :
Strength of rivets in bearing on 8 mm thick angle

Pb = crpf x d x t
= 250 x 21.S x 8
= 43000 N
=43 kN

Strength of rivet in single shear, ps


7t
= 't vf X 4 X d 2

= 80x 7t x21.5 2
4
= 29044 N
= 29.044 kN
Rivet value = 29.044 kN

Number of rivets required 56


= _
29 044

= 1.93 (say 2 rivets)


Member OC :

Strength of rivets in bearing on 10 mm thick angle

Pb = 250 x 21.S x 10

= 53750 N
= 53.75 kN
Strength of rivet in single shear

P8 = 80x7tx21.52
4

= 29044 N
= 29.04 kN
Rivet value = 29 .04 kN
- ~ of Steel stn1c tures l 2-171 Connecti
~
~

112
Number of rivets require
d= 29 _04

== 3.86 (say 4 ri'1ets)

Member AD :
Strength of rivets in bear
ing on 12 mm thick guss
et
Pb = 2s o x 21.s x 12

= 64500 N
== 64.5 kN

Strength of rivet in doub 2


le shear = 2 x : x21.5 x80

PS S == 58088 N

= 58.09 kN
·Rivet value == 58.09 kN

. . . 28 4-
Numb er of nvets required = - -18 7
58.09-

= l.67 (say 2 rivets)


For all rivets. adopt pitc
h= 3dn =3x 20 =60 m m

Edge distasnce = 2 d =
n 2 x 20 _
- 40 mm
i
i
J5 5 x 55 x a mm
. . - '!
i
.

!.

Gusset 12 mm th\ck
'
-·- ·• ·-· -·- · ··- ·-· T- · .
.

40 60 40 JL .1 10 x' 11 0x 10 m m

Fig. 2.11
pesign of Steel Structures [ 2-18 ] Connections
. ------- ----- ----- ----- ----- --;_ _--
.EX• 2.6 : Two plates of 12· mm thickness are to be connected by single riveted lap joint.
Using 20 mm diameter rivet, design the pitch. Take crat = 150 MPa, tvr = 100 MPa
and O'pr = 300 MPa.

Soln. : Gross diameter, d = dn +1.5

= 20 + 1.5 = 2.1 .5 mm
(i) To·find rivet value :
Strength of one rivet

7t 2
In single shear, p = -d X'tvf
s 4

7t 2
p = ~-21.5 xJOO
s 4

= 36305 N

In bearing, Pb = d X t X O' pf

Pb = 21.5 x 12 x 300

= 77400 N

Rivet value Rv = 36305 N (smaller of P5 and Pb)

(ii) To find pitch :


Strength of plate in tearing = (p-d) x t x crat
,Equating strength of plate to rivet value,

Rv = (p - d)xtxcrat

36305 = (p - 21,5) X 12 X 150

20.17 = (p - 21.5)

p = 41.67 mm ;( 2.5
dn = 2.5 x 20 = 50 mm

Adopt pitch of 50 mm c/c.


Dl.)si ,11 of Stee l Struc tun
;s 2- 19 ]
{B ) W E L D E D C O N N
E C T IO N S
• In troduction
When two structura l me
mb ers are jo in te d by me
an s of welds, the connec
a we lde d co nnection . A tion is
few decades ago designe calleq
rs had a feeling th at we
were less fatigue re sistan lded connectio
t an d th at a good-quality
welded connection could
negativ e fe elings had a gr no t be made. lheli! \
eat impact on the use of
welding in structures. Bu
in we ld ing equ ipment an t th e progress"" Se
d electrodes, the advanc ·•,ade
ing art and science of de
and the increasing tru st signing for Weldin
and acceptance of weld
ing have combined to m
impl ement fo r th e ex pa nd ake it .a powerr~
in g construction industry
. Further, the economics
he lp to offset increase in inherent in Welding
th e prices of material an
d cost of labour. In addi
pr oduction cycle made po tion, th e shortened
ssible by welding, have he
lped effect a quickening
cons truction. Welding wi in the pace of new
ll become increasingly im
portant as more people
depth of kn ow le dg e and acquire a greater
experience that goes with
it. Today most of the regu
and government de pa rtm latory agencies
en ts accept , in fact, pref
er welded jo in ts. There I
reasons for using a we ld are a number of
ed design, but a few basic
ones are the following:
1. Welded designs offer the
opportunity to achieve a mo
re efficient use of material
is th e only process th at s. Welding
produces a one piece co
nstruction.
2. The speed of fabrication
and erection helps compr
ess production schedules
3. Welding saves we ig ht an .
d consequently cuts costs
. Connecting steel plates
or eli m in ate d since th ey are reduced
often are not required.
4. No de du cti on s are th er e
for holes; thus the gross
section is effective in ca
5. Welded jo in ts are be tte r rrying loads.
for fatigue loads, impact
loads and vibrations.
Further, a properly we ld
ed jo in t is stronger than th
a rigid structure in co nt e jo in te d material. Fu se d
ra st to th e non-rigid struc joints create
tures made using other ty
co m pactness an d greater pes of joints. The
rig id ity of welded jo in ts
permits design assumptio
m or e ac cu ra te ly. ns to be realised

Welding offers th e be st
method for achieving a
rigid connection, resulting
be am de pth an d weight. in reduced
Th us , it noticeably lowers
the overall height of a bu
of th e structure an d cons ilding. The weight
equently the static loadin
g is con-siderably reduce
steel and re du ce s founda d. This saves column
tion requirements. Saving
in transportation, handlin
is pr op or tio na l to th e we g time and erection
ig ht savings.
Welding pe rm its ar ch ite
cts and structural engine
ers complete freedom of
design. The
pesign of Steel Sfructures [ 2-20 ] Connections
usage of outstanding design advancements such as open-web expan
ded (castel-lated) beams,
tapered beams, vierendeel trusses, cellular floor construction, tubular
column and trusses .are
a few examples of welded constructions (Plates 5 and 6).
TYPES
The basic types of welded joints can be classified depending upon the
type of weld, e.g.
fillet weld, groove weld (or butt weld), plug weld, slot weld, spot weld,
etc., position, e.g., flat
weld, horizontal weld, vertical weld and overhead weld, etc., and type
of joint, e.g., butt welded,
lap welded, tee welded and corner welded. These welds are show
n in Fig.

Fig. : Types and positions of welds


Groove and fillet welds are most frequently used. Groove welds, are
provided when the
members to be jointed are lined up. Various types ·of groove welds
are shown in Fig. Groove
welds requi re edge prepa ration and are thus costly. Single V, U, J,
etc., are cheaper to for~,
but require doub le the weld metal than double grooved joints. The
choice between single and
double grooves is usually a question of whether the higher cost
of preparation is offset by
saving in weld metal. Square butt welds are used for plates up to 8
mm thickness only. Fillet
welds are provided when two members to be jointed are in different
planes. This situation is
frequently met wit~,1n structures . Therefore, fillet welds are more com-
mon than groove welds.
Various types of fillet welds are shown in Fig.
Fil1et welds are normally easier to make, require less material prepa
ration, and are easier
to fit than groove welds. On the other hand, for a given amount of
weld material, they are not
as strong and they cause greater concentration of stress. In lightly
stressed structures where
stiffness rather than strength controls design and fatigue or brittle
fract ure is not a problem,
fillet welds are entirely adequate and generally more economical. Relia
ble fillet welds may be,
[ 2-21 1 Connectiolls
. f Steel StrUctures
Design ° b . .
e designed for severe stress and service conditions, ut, as a rule, gr00
and frequently ar ' d Vt
better in highly stresse struc tur es where smooth flow of stress is necessary. If th
welds are . t
. • t has the same characten.st'1cs as the parent metal, is finished sm oot h wit h it on both
butt JOtn . .
. d' the like) and has complete penetratio · ·h
sides (by proper gnn mg or , n wit no unwe ld zones it
approaches the condition of no joint at all and, for most common types of parent '
..
meta\, lllay
have impact and fatigue resista · t O that of the bas e metal.
nce equal or superio r . .

11 Square
9 I I Ii
Single · Double Single Double
vee vee u u

)
~
Single
w ~ 0
Double Single Double Single
.tee tee bevel · bevel J
Fig. : Types of groove welds
Slot and plug welds are used to supple
ment the ·fillet welds, wh en required
length of
fillet weld cannot be provided. Since, the
penetration of these welds cannot be asc
ertained and
since these are difficult to inspect, slo
t and plug are avoided. ·

Surface not
in.tension Surface in
tension

Convex Concave Mitre ·.

Fig. : Types of fillet welds

2.14 Definition of Welded Joint


• In these joints, various members of
the structures are connected together
by means of
weld, hence they are called welded join
ts.
2.15 Advantages of Welded Connect
ion s
1. As no holes are required for welding,
the structural members are. more effe
taking load. . ctive in

2. Welded joints are more stiff as compar


ed to riveted joints.
3. The overall weight of structural steel
is reduced by the use of welded joints.
-
pesign of Steel Structures

4. ·
s.
Appearance of welded joints is better.
[ 2-22]

No noise is produced in the welding process as in the riveting process.


Connections

6. Welded joints are often economical as less labour and material are required for a joint.

7. The speed of fabrication is higher in the welding process and requires less working space.
8. Any shape of joint can be made with ease.

2.16 Disadvantages of Welded Connections


1. Skilled labour and electricity are required for welding.
2. Internal stresses and warping are produced due to uneven heating an cooling.
3. Inaccuracies are caused by the deposition of weld metal in excess.
4. It is difficult to inspect a welded joint as compared to inspect a riveted joint.
s. Welded joints are more brittle and therefore their fatigue strength is less than the ivetedjoints.

2.17 Types of Welded Connections


Welds are of two types : .
1. Butt welds. 2. Fillet welds.

Reinfircement
Weld face
Fusion face

Root face -......i

---ii.-.
Reinfircement
Gap
Fig. 2.12 : Butt Weld

When members are allowed to lai, over each other, fillet welds are used. But welds are
used when the members to be connected are in the same plane.

Size = min. leg length

T
Leg Root ..,_,._ .,,....-
.--Throat thickness
.Original surface of work

l ·
-~v Toe
~~~~~--:F~u:.:s:--io....n---,
zone

Fig. 2.13 : Fillet Weld .


ffl
~
.
~ ~ ~ ~ ~ - - - - --[l2~~2~3JJ__ __ __ __c--c::~nnecti% 0
Design of Steel Structures
12' 1
'
0
60

i1
t~·=========r.L..♦-3----7i
'c: :7
r
+j 5 ~
Fig. 2.14 : A Typical Double V Butt Joint

1. Si11' of fillet weld (s) :


• The size of the normal fillet weld is taken equal to its minimum leg
length as·
shown in Fig. 2.13.
2. Throat thickness of fillet weld (t) :

• It is the perpendicular distance from the root of the fillet weld to the line joining
its toes.
• It is taken equal to k x size of the fillet.
• The value of k for different angles between fusion faces is given below in
Table ~-1.
• Almost in all cases right angle fillet welds are used for which k = 0.707
.
Table 2.1 : Values of k for Different Angles Between Fusion
Faces
Angles between 60° - 90° 91° - 100° 101 ° - 106° 107° - 113° 114° - 120°
fusion faces

Constant k 0.707 .0.65 0.60 0.55 0.50


t= k x siie of fillet weld
-· 0.707 X S

where , k = 0. 707 for angle between fusion faces 60° to 90°


(A most general case)
3. Effective length of fillet weld :
• The effective length of a fillet weld is taken equal to its actual length
mi~us
twice the weld size.

2.18 Symbols Used for Welds

• A. ~owledge of welding symbols is essential for a site engineer so as


to enable
him· tt). read the drawings.
• Symbols save lot of space in the drawing and as such descriptive notes can
be omitted,
• nn ~of~St~ee:::.,l...::S:.::tru.::.:c:..:.;tu::.re_s_ _ _ _ _ _---'-[-=2~-2:.. :4.. . .!]_ _ _ _ _ _ _ _ _C.: : .:,:on:: n:.:e:.:.c~ti.:. on:.;,;,. s
pes1g-:_
:;--- Table 2.2 : Basic Types of Welds and Their Symbols
---
t,_---
form of weld Section Symbol

..--
Fillet -
Jb ~
Square butt . i I r ll
- - ·-·-· , 0
. 60 min. --

Single-V-butt
,· ~
1· 0.7to3mm V
) • t (
--1 i4--2 to ts mm
~
, 0 , ,

Oouble·V-butt
Oto15mm~ ;0.7to3mm
. I *% 60 min.

~ ~
_ 3mm
I
0.7to3mm
X
Single-U-butt i
• r u
R
--
Double-U-butt . )
I ,,
( ,,

Single-bevel-butt __
l , ~-~ ,' ( , ,
t>
Double-bevel-butt
,
! t
-
J g , .

Single-J-butt -
! ~ ··
( 1J
Double-J-butt
I C -~
R
2.19 Permissible strc"i~ and strength of fillet weld

• Strength of fillet wcl<l is taken equal to the resistance ottered by it against shear:
• It is so, because it is weak in shear as compared to other modes o'r failure.
p == Pq X / X t
where, P == Strength of the joint
Pq == Permissible shear stress on section through
throat of fillet weld.
As per IS 816 - 1969, its value is taken as 108 MP a.
I == Effective length of the weld
t = Throat thickness of fillet weld.
~D~es~·ig~n~o~f:JS~t~ee:!_l~S'.!!tru~c~tt~l
f~es~_ _ _ _ _ _J_
[_ :2 ~ -2 :: 5' .. .. .] L -- --
2.20 Design of welded -- -- -- -: :. ~
joints
Following specifications ·
• as per IS 816 - 1969 shall ~
be observe d.m t he desig
welds. n of{'
tl\\t
11.
Size of fillet weld :
'
Table 2.3 : Minimum Siz
e of Fillet Weld
Thickness of thicker
part M in im um size
Upto and including 10 mm
3 mm
Over 10 mm upto and in
cluding 20 mm 5 mm
Over 20 mm upto and in
cluding 32 mm 6m m
Over 32 mm upto and in
cluding 50 mm
10 mm
• Maximum size :
(a) Maximum size of fillet we
ld applied to a square edge
of a plate should not exceeo
thickness of the edge m
inus 1.5 mm.
(b) Maximum size of fillet we
ld applied to rounded toe
of rolted steel section sho
not exceed 3/4 x the th u\o
ickness of rounded toe.
2. Overlap : The overlap
in a lap joint should not
be less than 5 times the
thinner plate as shown thicknesso!
in Fig. 2.15.
l t .
~ l- - ,- - - - - - - - - - - -
t . I~- - - - ----i-J
---
--.
----!Lap .,, St \+-
Fig. 2.15
3. Side fillet : In a lap jo in
t made by a side or long
itudinal fillet weld, the len
fillet weld should not be gth of eac~
less than the perpendicu
lar distance between the
m.
t- .1 1. -

T
d

1 h T T rn T n T n m
-- -- -- -- -- -- -- -. J
L = Not less than d
d =N ot to exceed 16 t
Side fillet F'Ig• 2•16
[ 2-26 ] Connections
pesign of Steel Structures
---
4, End Return : Fillet welds terminating at ends or
sides of parts or members should
ers for a distance not less than
preferably be returned continuously around the corn
twice the weld size as shown in Fig.2.17.

End returns not less than twice weld size


Fig. 2.17
n used, should not be subjected to
5, Single fillet weld : A single line of fillet weld, whe
bending about its longitudinal axis .
_ _ _ _, ..
._ _ _ __..&_n_,_ _ _ _ _rmr-_ _
.,_ r:: : 110.P

Correct

--- --- -'d llW ll~ I-- -- -,. -- -- -- -- 1 ..


~'-,-- -- -- -- -- -- - Incorrect
Fig. 2.18

SOLVED EXAMPLES
connect a tie bar 80 mm x 10
Ex. 2.7 Calculate the strength of 8 mm fillet weld used to
mm as shown in Fig. 2.19.
Take Pq = 108.0 N/mm 2 •

•~ - FIiter weld size 8 mm

Tie bar 80 mm x 10 mm

Fig. 2.19
00 = 280 mm
Soln. : Effective length of weld, l = 100 + 80 + I
Throat thickness, t = 0.7 x s = 0.7 x 8 = 5.6 mm
Strength, P= pq x/x t
·g~n~o~f~S~te~e:!._l~S~tru~c~tu~r~es~_ _ _ _ _ _J[L2~..:::2.!. 7.1]__________C~onnect1·
~
D~s~ -~
= 108 X 280 X 5.6
= 169344 N
= 169.34 kN
Ex. 2.8 : Design a suitable filled weld to connect a tie bar 80 mm x 8 mm at its e ~ ·
a gusset plate 8 mm thick. Take the permissible stress in tension to be 150 MP·
. a.
Soln. : The joint is to be designed for the full strength of the tie bar.

Strength of tie bar = crat x bx t


= 150 x 80 x 8 = 96000 N = 96 kN
Maximum size of weld = 8 - 1.5 = 6.5 mm
Using a 6 . mm fillet weld ' .

Strength of the weld/mm length


= Pq xix t
= 108 x 1 x 0.7 x 6 = 453.6 N
Length of the weld required

= Strength of tie bar


Strength of weld/mm length

96000
= 453.6 = 211.64 mm say 220 mm

Provide the weld length as shown in Fig 2 20 It satisfi th 'fi •


· · · · ies e spec1 1cations.
1. Length of each side weld
= llO mm> 80 mm, the transverse distance between welds.
2. Transverse distance between side welds
= .so mm is less than 16 t i.e. 128 mm.

· · Size of fillet =6 mm
T
80mm

Tie bar 80 mm x a mm
1
Gusset olate a mm thick
Fig. 2.20
-
gn~o~f:=S-=tre=e~l~Si:tn~1-:c=tu~r~es~~~=~i:;F~~~~i::=2
~-s~j~ [~2::--=2:::8::]::::::=:===-=-=-=-=========C=o=n=n=ec=t=io~n=s
~ 2.9 : Design a fillet welded lap joint between two plates of size 150 mm x 8 mm and
• •
150 mm x 12 mm as shown in Fig · 2 ·21 · Th e perm1ss1b · •
le stresses for plate m
tension and weld are 150 MPa and I 08. MPa respect'1ve1y.

12 mm~
_-~r---:- ----AW®
L----n mr--=
l mm _ .
=:::1,- ..Ta
\__ mu
t
Fig. 2.21

Solo. : The weld will be designed to develop full strength of the thinner plate. Strength of.the
plate of size 150 mm x 8 mm

= crat x bx t =150 x 150 x 8


= 180,000 N
= 180 kN
Both plates will elongate under the load and therefore stress will be equal in both plates.
Equal stress in both plates means that the thicker plate will carry more force as :

Force
Stress = .
Cross-sectional area

Hence provide the weld size proportional to the thickness of the plates. Let SA and SB

be the size of weld for plates A and B respectively.


SA 12 ... (i)
-S = - 1.e. SA =1.5 SB
B 8
Total strength of weld = 108 x 150 x 0.7 x SA+ 108 x 150 x 0.7 x Sa
=11340 (SA+ Sa)
Equating,
Strength of weld = Strength of thinner plate

11340 (SA+ Sa) = 180,000


...(ii)
= 180,000 = 15.873
SA +Sa 11340
- l.5SB + Sn = 15.873
s8 = 6.35 mm say 6.5 mm
SA = 1.5 x 6.35 = 9.52 say 9.5 mm
These sizes are smaller than the thickne ss of the connec ted plates at least by l.s Ill
and hence the design is all-right. .tii,
Ex. 2.10 : A 90 x 60 x 6 mm angle is require
d to be connec ted to a gusset plate by w .
of 6 mm size. Permis sible shear stress in the weld is 108.5 N/mm 2• Calculate1ding
lengths of weld. For 90 x 60 x 6 ISA, C = 13.9 mm, Cxx = 28.7 mm, A-e the
2 xx - 865
mm • Design the joint for maximum force.
Soln. : Cross-s ectiona l area of ISA 90 . x 60 x 6 mm, A = 865 mm 2 •

Maxim um force in angle = crat x A= 150 x 865 = 129750 N


= 129.75 kN acting along e.g. of section
Size of weld = 6 mm
Effective length of weld :
Total effectiv e length of weld required

129.75x 103
= ---- 284.73 mm(sa y 290 mm)
108.5x 0.7x6

The distribu tion of the weld should be such that the, e.g. centre of gravity of the weld
coincid es with the line of action of the forces.

'
p I

I
I
·- ·-·-· : - ·-·- ·-•- •. ·. ·- ·r·-·-·- ·- .- .
p

I 61.3 mm
.
28.7 mm
. it
·\ ·

,,,,,:.J

Fig. 2.22
From Fig. 2.22, the total effective length of weld
x 1 + X2 + 90 = 290 mm
x 1 +x 2 = 200 mm ')
, • •( I
esign of Steel Structures Connections

Force in weld per mm length


= Pq x 1x t = 108.5 x 1 x0.7 x6 = 455.7 N /mm

Taking moment of forces about the bottom edge of member,

90
455.7 X Xi X 90 + 455.7 X 90 X = 129750 X 28.7
2
Xi+ 45 = 90.79 .... (ii)

xi = 45.79 say 50 mm

Putting value of x 1 in (i)


I

and x2 = 154.21 say 155 mm


Ex.2.11 Calculate the size of fillet weld required for joint shown in Fig. 2.23 to carry an
axial force of 200 kN.

. .
I
I
I
I ·

---'t -•---- --\, ,-•


m a---
10
200kN
200 kN

Fig. 2.23

Soln. : Take Pq = 108 MPa

p = Pq xLxt
Using

3
P 200x 10
--=
t = Pq xL 108(2 xl00+ 100)

= 6.17 mm
But t = 0.7 s
0.7s = 6.17
s = 8.82 mm say 9 mm.
!2D~es~ig~n~o~f~S~te=c!__I~St~n~1c~tu~rc~s_ _ _ _
_ _ _1..[2=--..: 3.:. .l2]_ _
_ _ _ _ _ _ _~C on ne cr
1
Important Points lo~1

(A) RIVETED CONNECTIONS I


• A piece of round steel forged in place
to connect two or more than two stee
l
together is known as rivet. 111enibe1
~\
• A rivet consists of a 'head' and 'sh
ank' .
• Rivets are classified as :
(a) (i) Hot driven rivets
(ii) Cold driven rivets
(b) (i) Shop rivets
(ii) Field rivets
(c) (i) Power-driven rivets
(ii) Hand-driven rivets
• Types of Riveted joints are :
(i) Lap joints
(ii) Butt joints
• Effective diameter of rivet
d = dn + l .5 mm for rivet dia < 25
mm
d = dn + 2.0 mm for rivet dia > 25
-
mm
• Pitch of rivets
(i) Min. pitch = 2.5 dn
(ii) Max. pitch = 32 t or 300 mm whicheve
r is less for any two adjacent rivets.
• Edge distance 2 dn=
• Failure of riveted joints occur due to
(i) Tearing of plate
(ii) Sharing of rivet
(iii) Bearing failure of plate
(iv) Bearing failure of rivet
(v) Edge cracking of splitting failure
• Rivet value
(a) For single shear - Minimum of

_. l
Conn ectio ns
Design of Steel Struct ures [ 2-32 ]

(i) Pb = CT pf X d X t

. 7t
(ii) ps = tvf x 4 xd2

(b) For doub le shear - Minim um of

(i) Pb=c rprx dxf pu: -


., ~!l~~w~~8
7t 2
(ii) Pss =2x1 :vf x-x d
4

• Stren gth of rivete d point - minim um of

(i) Stren gth of plate = crat (p - d) x t

(ii) Rivet value

• For powe r drive n shop rivets

tvf = 100 MPa and crpf = 300M Pa

• For powe r drive n field rivets

tvr = 90M Pa and crpf = 270M P~

• For Hand drive n rivets

• tvf = 80 MPa and cr pf = 250 MPa

Stren gth plate weak ened by rivet hole x 100


• Effic iency of rivete d joint = Strength of solid plate

I (B) WELDED JOINTS I


by mean s of weld,
• When vario us memb ers for the struct ures are conne cted together
they are called welde d joints .
:
• Welded joints are more advan tageo us than rivete d joints due to
(i) Avoidance of rivet ho les
(ii) More stiffn ess
~~~~~~~----.-=-=~-=3:]:---------ccConnecr
. ■

Desi gn of Steel Structures ····-,:~.%~~


(iii) Reduction in overall weight of structure

(iv) Better appearance


(v) Reduction in noise

• Types of welded connections are :


(i) Butt welds
(ii) Fillet welds

• Throat thickness of filler weld is generally

jt=0.707xsl

where, s = size of weld


• Permissible shear stress in throat of fillet weld ==
108 MPa .
• Strength of the joint

\P=pq x/x tl

Exercise Problems

·(A) Riveted Connections I


1. Define 'rivet value' . How and why is it calculated
?
2. Explain how pitch is decided for riveted joints.
3. State two/four modes of failure of riv,. ~ed joints.
4. Define "tacking rivet". State its function.
5. Define pitch distance and gauge distance of rivets.
6. State LS. specification of minimum pitch of rivets.
7. State LS. specification for diameter of rivet hole.
State the diameter of rivet hole for
25 mm diameter rivet.
8. Draw typical plan and section elevations of double . I
rivetted lap joint and single cove
double riveted butt joint.
9. Define tacking rivets. State functions of tacking rive
ts.
10. What will be the hole diameter, if the nominal diam
eter of rivet is
(i) 20 mm (ii) 28 mm and (iii) 25 mm ?
Design of Steel Strocture s [ 2-34 ] Connec tions

11 . Find the value of 16 mm power driven shop rivet in a lap joint The thickness of the plate
is 10 mm. Use I.S. specific ations.

12. l t es of
Calculate the value of a 30 mm diameter rivet in a lap J·o·mt connec t·mg. th e pa
thickness 20 mm and 25 mm. Given t yf = 100 N/mm2 , 0 P = 300 N/mm. 2

13. Find out the number of power driven field rivets required to connect the tension member
in a roof truss to a gusset plate 8 mm thick. The tension member is a flat of size 100
mm x 8 mm thick is subjecte d to a pull of 52 kN. Use 16 mm diameter rivets.
14. Fig. 2.24 shows a joint in the lower cord of a fink roof truss.
Find out the number of 16 mm, power driven shop rivets at the end of the members
U I L 1 U 2 L 1 and also those for the lower chord.
Use IS specific ations.
70kN 70 kN.

--~,'...,....,/i,-- GUSSET PLATE 10 mm THICK

5
2L 125 x 75 x 8

490 kN ·-•- ·- ·-·-·- ·- · -·-·- ·- ·- ·-· _ -· - ·- · \.,·(_ ___·- -·-·-· -·- ·- ·-· .- ·- ·-·- ._,,. _4_20..,..kN
Lo'==========::::d::= ==--==-..-.-==-=-===-=-==-=t L2
Fig. 2.24

15. A double riveted lap joint is used to connect 12 mm thick plates. Calculate strength and
efficiency of joint if 20 mm diameter rivets are provided at a pitch of 60 mm. Allowable
stresses in tension, shear and bearing are 150 MPa, 100 MPa and 300 MPa respectiv~ly.

16. A double riveted double cover butt joint is used to connect 10 mm thick plates. If 22
mm diamete r rivets are provided at a pitch of 75 mm, calculate strength and efficiency
of the joint Al !owable stresses in tension, shear and bearing are 150 MPa, I 00 MPa
and 300 j\·: :~a respectively.

17 • A single riveted double cover butt joint is used to connect 10 mm thick plates. Calculate
strength and efficien cy of joint if 16 mm diameter rivets are provided at a pitch of 75
mm. Allowable stresses in shear, tension and bearing are 100 MPa, 150 MPa and 300
MPa respectively.
I
s I Stmctures [ 2-35 ·] Conne .
Design of tee
d to 90 MPa, 270 MP Ct10~I\
Solve above Ex. I 6, if allowable stresses are change
18. a and 1~!
. I
MPa, in shear, bearing and tension respective Y· .
1

The plates of 20 mm thick are connected usmg • ·ogle cover double nveted b
19 •
si Utt Joint
. . h Of 100 mm. Calculate strength of th . ·
20 mm diameter rivets are provided at a pitc
eJoint
and efficiency. Take O"at = 150MPa,
crpf =300 MPa and -rvf =lOOMPa.

20. Two plates 20 mm thick are to be connecte d by single rivetedd fidoub


.
le cover b .
Utt Join
using 20 mm diameter rivets. Calculate minimum pitch require or its
greatest strengt~
Also determine efficiency. Take O"at = 150MPa, 'tvf = lOOMPa, <>pf = 300MPa.

21. A single riveted lap joint is used to connect plates 10 mm th ick. If 20 mm


diameter riYet
are used at 55 mm gauge, determine the strength of joint
Take:
Working stress in shear in rivets = 80 MPa
Working stress in bearing in rivets = 250 MPa
Working stress in axial tension in plates = 156 MPa
22. Design a lap joint to carry a load of 350 kN. The rivets are 20 mm in diame
ter and place
in a double row. The permissible tensile stress in the plate is 150 MPa.
The permissible
bearing and shearing stresses for the rivets are 246 MPa and 100 MPa
respectively.
23. A single riveted lap joint is used to connect plates 10 mm thick. If 16
mm diameter
rivets are used at a pitch of 50 mm, determine the strength of joint. Assum
e pds rivets.
24. 16 mm thick plates are joined by double layer butt joint using 10 mm thick
cover plates
Calculate the strength and efficiency of the joint per pitch of 90 mm, if
20 mm diameter
power driven shop rivets are used. Permissible stress in tension is
150 MPa, shear
stress in rivets is l 00 MPa and bearing stress in rivets is 300 MPa.
25. Calculate the minimum pitch to be provided to connect 16 mm thick plate
using 20 mm
diameter rivets. Double riveted lap joint is to be provided, permissible
stress in tension
is 150 MPa, shear stress in rivet is 100 MPa and bearing stress in rivets
is 300 MPa,
26. 12 mm thick plates are connected using double riveted lap joint using
16 mm diameter
rivets at a pitch of 80 mm. Calculate strength and efficiency of joint.

Take crat =140 MPa, cr pf =270 MPa and 'tvf = 90 MPa.


27. Two 150 mm wide and 16 mm thick plates a~e connected using single
cover double
_..I
-
p csign of Steel Structures [ 2-36] Connections
riveted butt joint, 20 mm rivets are provided at a pitch 120 mm. Calculate the strength
of joint. Permissible stress in tension is 150 MPa, shear stress in rivet is 100 MPa and
bearing stress in rivet is 300 .MPa.

I (B) WELDED JOINTS I


28. State the advantages and disadvantages of welded connections over connections. Write
any four points of differences.
29. Define effective length of weld. State how to calculate length of weld required.
30. State the relation between throat thickness and size of fillet weld.
31. State any two types of butt weld.
32. Draw a neat sketch of welded single 'V' butt joint.
33 . Draw a neat sketch of double ' V' butt joint in weld.
34. State two types of welded joints.
35. Drawing a neat sketch show the size and thpat of a welded lap joint. What is the
relation between them ?
36. Find out the strength of the welded joint as shown in Fig. 2.25. Take Pq = 108 MPa.

Fillet weld size = 5 mm

Plate 150 x 8 mm

1
'1aom~
Fig. 2.25

37. Design a fi11 ct welded joint between two plates of size 200 x 10 mm and 250 x 10 mm.
38. Design a fi l! welded lap joint between two plates of size 200 mm x 8 mm and 200
mm x 16 m '· as shown in Fig. 2.26. Take the permissible tensile stress in plates and
strength of wdd as 150 MPa and 108.5 t\1Pa respectively.
Plate 200 mm x 16 mm ·

p
p
Sa
·p1ate 200 mm x 8 mm
Fig. 2.26
Design of Steel Structures [ 2-37 ] Co n ~
A tie in a truss consists of pair of angles ISA 90 x 60 x
39. 10 mm welded on either ·.
. Stde
of a gusset plate 12 mm thick thro ugh the longer legs.
De gn th e weld ed joint if the
s1
permissible stresses in angles and fillet welds are 150 MPa
and l 08 MPa respectively
End welds are to be provided. Use 7 mm size fillet weld
s.
I
40. Calculate the length of weld requ ired to connect an ISA
100 x 100 x 10 with gusset
plate using 8 mm fillet weld as shown in Fig. 2.27. The
angle is subj ecte d to an axial
force of 100 kN. Allowable shear stress in weld is 110
MPa . Cxx = CYY for angle is
28.4 mm. How ever design the section for full strength.

" ISA 100 x 100 x 10


··-·-·-·-· ·-·-·-·-·-·-·-·-·-·-·-·-·-·-·-·-·-·-·-·-·-· ·-·-···-·-·-·-··- l_______ ·..·.-·-·-·-·-·-·.·-·-·-
. mr"Jm?fflrffn;:p== ===r== == = 28.4 mm . fi?.;Ziiill• .
T
Fig. 2.27
41. A flat 120 x 10 mm is to be connected to a gusset plate usin
g 6 mm fillet weld as shown
in Fig. 2.28. Calculate the minimum lap required to full stren
gth of flat. Allowable tensile
stress in plate is 150 MPa and shear stress in weld is
11 0 MPa .

.&
I,..
I
X-, -.

.. .



'
T120
•• ••
·,
r
I

J_
Fig. 2.28
42. Calc ulate mini mum lap required to conn ect a flat 100
x 16 mm with guss et plate usini

T~ --f1 6t,.-

100 a- --- -~ --- --- --- :r- --- l--


'-- -
1 ~' 77 77 77 7, ~~ --- -J
Fig. 2.29
Design of Steel Stiuctures [ 2-38 ] Connections
Io mm fillet weld as shown in Fig. 2.29 if it is subjected to force equal to its full strength
in tension. Allowable shear stress in weld is l 00 MPa and allowable tensile stress in
flat is 15 0 MP a.
Calculate the size of weld required for the joint shown in Fig. 2.30. Allowable stress
in weld is 100 MPa.

100 kN
10 ·-·-·-·-·-·-·-·-·-·-·- ·-·-·-·-·----

Fig. 2.30
44. Enlist any four advantages of welded connection. Draw a neat labelled sketch of a fillet
weld. Find safe load transmitted by a fillet welded joint between a flat 60 mm wide
overlapping 100 mm over a gusset plate. Thickness of both plates is 10 mm. Weld is
on all sides of overlap. Size of weld is 6 mm.
45. A tie member 75 mm x 8 mm has to transmit an axial load of 90 kN. Design the fillet
weld and calculate the necessary overlap. Consider the welding on all four sides.
46. A 100 mm wide and 12 mm thick tie plate has been connected to a gusset plate by a
10 mm fillet weld. The lap of the plate over gusset plate is 120 mm. Fillet welding has
been done on two longitudinal edges and not at ends. Find out the strength of the fillet
welded joint if permissible shear stress in weld is 102.5 MPa.
47. A tie in a truss consists of a pair of angles ISA 50 x 50 x 6 mm welded on either side
of a 8 mm thick gusset plate. Design the welded joint, if the permissible stress in fillet
weld is I 08 MPa.
48. A plate of 8 mm x 60 mm in cross section is connected to a gusset plate. What is the
length of weld required for full strength of plate if size is weld is 6 mm ? Assume
stresses.
49
· Find length of 8 mm thick fillet weld to connect a tie 100 mm x 12 mm to a 16 mm
thick gusset plate. The permissible stresses in the 'tie bar and fillet weld are 150 MPa
and 108 MPa respectively.
50.
Design a suitable fillet weld to connect a tie bar 80 x 10 mm to a 12 mm thick gusset
I
[ 2-39 ] Coh ....1
Design of Steel Structures ·~ ecti
weld are 150 MPa and Ot.
he penn issib ility stres s in the tie bar and fillet
pa
It e. T . lOg ~~
.
respectively. . 15A 1 00 x 100 x I 0
51.
Calculate length of fillet weld requ ired to conn ect an
. . lllltl Wi;
.
gusset plate using 6 mm weld as show n m Fig. 2 . . • 31 The angl e ts subje cted to I
an axi~
force O 120 kN. Take allow able shea r stres s m · Id as 100 MPa . C == C fi
we xx Y'f or angJ;

is 28.4 mm

I
r--~;HN~O I
I
I I

'7
I

. : 120 kN
• Y" - •
',,
• ~•-• - •- • ~•-• -•~• -•-• -·-• -•-• -I1
••
-·-·-·-·-·-
I 28.4
,' I
I
, ,,,,,,

r--x2 -- ISA 100 x 100 x 10

Fig. 2.31
52. Desi gn a suita ble fillet weld to conn ect a tie bar 100 x 12 mm
to a 16 mm thick gusset
plate. The perm issib le stresses in the tie bar and fillet weld
are 15 0 MPa and 108 MPa
respectively.
53. Calc ulate the size of fillet weld required for joint show n in
Fig. 2.32 to carry an axial
load of 200 kN.

T~
100 mm :- ·-·- · - · - · -·-·-·-:-...,._~► 200 kN
l ~'il 777 77, r--- --1
Fig. 2.32
54. Calc ulate mini mum pitch to be provided to conn ect 12
. •. . h' k · 11 0 Jll(ll
mm t 1c, plate s using .
diam eter nvet s. Doub le nvet ed lap joint is to be prov ided.
Take crat =150 MP~ , 'tvr =100 MPa , crpf ==300 MPa .
55. Calc ulate size ~f fillet weld requi red to connect fl t . h
1 . in fig. I
2.33. Take 'tvf =100 MPa . a a wit guss et plate <! $ shown I
.

J
[ 2-4 0] Co nn ect ion s
Design of Steel Structures

T~ . 160 kN
·-·- · - · -·-·-·""·-·-·-·- · - · - · -·- · -..,....---t►
1
1·00 mm
.
Fig. 2.33 .
2.34.
ted -with a gu sse t pla te usi ng 6 mm fillet weld as shown in Fig.
56. A flat is con nec
gth 'x' to car ry a for ce of 100 kN. Take tvf for weld as I Ob
Calculate mi nim um lap len
MP a.
X .
.
. ,
.

-+1101+- .

100

Fig. 2.34

□□□
Chapter

I Design of Tension
Membe!J

In tr od uc tio n

• A structural member subje


cted to two pulling (tensile
) forces ap pl ied at its ends
a tension member. Th e me is called
mbers and connections are
the connection and bending so ar ra ng ed th at eccen-tricity in
stresses on th e member are
no t de ve lo pe d. Also, the
mo me nt /st re ss es due to se bending
lf-weight of the member,
be in g in sig ni fic an t, are ne
Ho we ve r, if so me eccentr glected.
icity exists due to either me
or ec ce nt ric ity in connectio mb er no t be in g perfectly str
ns, then either bending str aight
esses are co n- sid er ed in the
or net ar ea is modified as design
per specifications. The stren
gth/effi-ciency of a tensio
ma y be seriously affected n member
because of the end connec
tions (b ol t holes), sh ea r lag
of lo ad s ( ca us in g bu ck lin , reversal
g) and, be nd in g moment
co nn ec tio ns or lateral loa s (d ue to ec ce nt ric ity in
ds on the member). the end
• Th eo re tic all y, a me mb er
subjected only to axial ten
sion is su pp os ed to be mo
an d ec on om ica l structur st efficient
al element. It is because
the me mb er do es no t bu
or ov er all up to and be yo ckle locally
nd yield, and a~ o because
th e en tir e cr os s se cti on is
to un ifo rm stress. Fo r du subjected
ctile materials, a uniform
str es s di str ib u-tio n assum
re as on ab le wh en th e ma ption is
terial is loaded beyond the
co nc en tra tio n ne ar th e ho yi eld str es s. Al th ou gh the stress
les leads to the yi eld in g
of ne ar by fib er s bu t th e
th e steel pe rm its redistrib ductility of
ution of overstress in the ad
joining sec-tions till th e fib
from ho les pr og re ss iv ely ers away
reach yield stress. Theref
to as su me a un ifo rm str or e, at ul tim ate load, it is reasonab
ess distribution. le
• Th e str es s in an ax ial ly loa
dded tension member can
be de ter mi ne d by dividing
lo ad wi th its cr os s-s ec tio the axial
na l area. Of ten tension me
mbers are co nn ec ted at the
wi th bo lts . Th e pr es en ce ir ends
of bolt ho les influence th
ho le or ho les . Fo r ex am pl e stress at ~ cr os s section
e th e tension member of thro~gh
Fi g. is a 150 x 8 m m flat
to a gu ss et pl ate . Th e cro connected
ss-sect1.0na1 area of th e fla
t/bar at se cti on a-a is 12
00 mrn 2
Design of Steel Structures [ 3-2 ] Design of Tension Member
960 mm 2 [(150-2 x 15) x 8] and thu
s section
8), while at sec tion b-b it is onl y
(150 x
hig hly stre sse d. Th is red uce d area at section b-b is referred to as net
b-b wi II be more le
the red uce d sec tion is cal led net section. The unreduced area, for examp
area and
culated
are a at sec tion a-a is the gro ss are a. Stresses in a tension member are cal
the
min imu m net cro ss- sec tion al area ava ilable. To maximize the available
on the basis of
in a bol ted con nec tion , the bol ts are placed in a single line. Often the length
net area
more than one Jine.
of the connection necessitates using

Gusset plate
15mm l /• . 150mm •/

~-- t--d iam ete r e -


holes T ·Section a-a
b b 15 mm
~ t'+-
r
a · --,, ...,c :-- a
150 !SF 8
• -·
Gross area and net ·area
Section b-b

3.1 Definition
mb er car ryi ng dir ect ten sion is called a tie or tension member.
• A me
ers , bridges and bracing systems.
ber s are fou n d· in
· roo f trus ses, tow
,
• on mem
• 1 ens 1
'T'

3.2 Types of Se cti on s Used


t ·on s wh ich are used at tie members are shown in Fig. 3.1
The different forms of sec
1 ·
• me mb er for bra cing buildings and timber trusses.
. ry rod - n os 11. ,f tly u~~ed as tie
(a) Ordma
.
. . . om mo n 1Y used as a tie bar. in light trusses.
(b) Flat - c
. . Us "'d 1:0 single
plane trusses.
(c) Sm g 1e :. ~.g 1e- v

plane trusses.
(d) Double angle - Used in single
.on - Used in single plane trusses. . .
(e) T-sect1 resist bendmg also .
• _ Single plane trusses where it has to
(f) Double channel section
. angle sec(on 1 s _ Lig
ht double p]ane riveted trusses.
(g) and (h) Multi-
. Eye bar - Use d m . pin connected trusses.
(1)
1

Design of Steel Structures [ 3-3 ] Design of Tension Me~


•Y
1 ly

il · .::1r1:
· --·:•
- X·
I
.
X·-·- ·-·-X X·-· _ , ___ X · x-- ·-· i .
j
. .
.
. i
tY !Y . .iy·
(a) (b) (d) .
(c)

.,y.
i I
'l .;
i . . i .
i

F
i
. ; - .

➔·-·-·-·- ·
l ~ ·
.i '
X·-·-·-· · : , ·-·- ·- X X·· _:~
i . .
.. ·.ry i

(e)
y ·'
. (f) (g)

(h) . (I) ·
Fig. 3.1 : Various ·Forms of Tension Memb ers

3.3 Stren gth of Tens ion Mem bers


• Two limit states are established for a tension member; one is based on the yield
strength
of the member due to its excessive elongation, and the other is based on the
ultimate
strength of the member. Ultimate strength is usually critical at any point where
the area
of the member is reduced (e.g. due to the presence of bolt holes).
• A ductile steel tension member without holes can resist without fracture a load
larger
than its gross sectional area times its yield stress because of strain hardening.
However,
a tension . member loaded until strai.n hardening is reached will elongate excess
ively
before fracture thereby making the member unserviceable and may even cause
its failure.
This implies that the yielding of the gross section of tension member causes
excessive
elongation and therefore the load corresponding to the yielding of gross sectio
n may be
taken as a limit state.

• On the other hand, a tension member with the bolt holes a.t the ends of the
mem-ber
may be subjected to tensile stresses well in excess of yield stress to as high as
ultimate
stress without the memb er suffering excessive elongation. This is due to the
help given
by the supposedly non-active metal behind the hole parallel to the di rec-ti on of
pull. Just
--
vesign of Steel Structures [ 3-4 ] Design of Tension Member
prior to breaking, the metal tends to narrow at that point where the fracture occurs, and
the surplus metal behind the hole restricts this contraction of the strained metal and tJms
increases the ultimate strength. Here, the member can fail by fracture at the net section
through the holes. This failure load may be smaller than the load required to yield the
gross section away from the holes. Hence, the fracture strength of the net section
through the bolt holes at the member ends is considered another limit state.
• It is to be realised that the portion of the member with holes is small part of the total
length of the member. Though the strain hardening situation is quickly reached at the
net section portion of the member, yielding there may not really be a limit state of
significance. The overall change in the length of the member due to yielding in this small
part of the member length may be negligible.
The design strength of a tension member, therefore, is the smaller of the foJlowing two
limit states.
1. Limit state of yielding in the gross section-intende d to prevent excessive elongation
of the member. Thus, to prevent excessive deformation the stress on t~e gross
section must be less than yield stress f y .
2. Limit state of fracture (also called rupture) in the net section-where bolts or
rivets hole are present. To prevent fracture, the stress on the net section must be
less than the tensile strength.
• Exceedance of the yield limit state will require a -significant design overload and
accompanying large deformation will usually forewarn occupants. Fracture through holes, .
on the other hand, wi11 occasion a sudden failure, and it is for this reason that the
designer must consider carefully the impact of holes on the behaviour of a tension
member.
• The tension members are connected by rivets or welds .
• When they ar · connected by rivets, the cross-section of the member is weakenes_by
the rivet ho]F :rilled in it.
• Even though the rivet hole is plugged by rivet itself while resisting the tensile load, the
member and the rivet get separated because they are only press-fitted and there is no
perfect bond between the member and the rivet. Hence the effective area which resists
the tensile load gets reduced .
• Therefore the full cross-sectional area of the member minus the area of rivet hole is
to be considered and it is called as net sectional area.

brr
Design of Steel Structures ( 3-5 ] Desi of Tension Mernber

• In case of welded joints, the full cross-sectional area is available to resist the load.
• The load resisted by tension member is product of net sectional area and the permissible

stress i.e. IPat = crat x Anet I


3.4 Permissible Stress in Axial Tension
• The direct stress in axial tension on the effective net sectional area should not exceed
crat'
crat = 0.6 fy
where, crat = Permissible stress in axial tension.
fy = Minimum yield stress of steel in MPa.
If fy = 250 MPa

crat = 0.6 fy =0.6 x 250


crat = 150 MPa

3.5 Net Sectional Area


Case 1 : For single angle connected by one leg only

where, A1 = Net cross-sectional area of connected let.

A2 = Gross cross-sectional area of unconnected leg


3A 1
and k= 3A +A
1 2

= Reduction factor for outstanding leg

Slngle angle

-*
Gusset plate
Fig. 3.2
Design of Steel Structures [ 3-6 ] Design of Tension Member
Case 2 : For pair of angl es Paced
1 back-to-back (or a single tee) connected by
one leg of each angle (or by flange of tee) to the same side of a gusset :

i
l

t
I
____.._1
I T\.
Single tee

i
Guaet
plate
Fig. 3.3
Anet = A1 +A2 ·k
where, A1 = Net sectional area of connected legs (or flange of tee)
A2 = Area of outstanding legs (or web of tee)

For double angles,

Free tee section, A1 = tr (bf - 2d)

Case _3 : For double angles or tees placed back-to-back and connected to each
side of a gusset or to each side of a rolled section carrying direct tension :
A = gross area - deduction for holes break = 2 (Ag - d x t) _
net
provided that the angles or the tees are tack riveted along their length at a pitch not exceeding
1m. When two angles or tees placed back to back are not tack riveted, the provisions under
Case 2 and 3 above do not apply and each angle or tee acts as a single angk or tee connected
to one side of the gusset and acco rd ingly the net area is determined as per case I.
r
~D'.::es~ig~n~of~S~te~el~Stn~1c~tt~1r~es~ _
_ _ _ _ __J[]~_??J_
]_ _ _ __:::.
3 D::.e.:.:
si:.!ii:g~n_o_f_T_e_n_s_io_n_
M-.:.:~
~air of ano
on both sides dfles
gusset Main rivets at Joint

-•:► ~--+-----+- ■ ---+··


I

Gusset plate Tanking rivets

Pair of tees on
on both sides
of gusset

-+-·- +----+-·

-+----+-~--+-· .

Gusset plate Tacking rivets


· Main rivets
at joint
Fig. 3.4

I SOLVED EXAMPLES \
Ex. 3.1 Calculate the strength of ISA
90 x 60 x 8 mm used u~ a tie
me mb er with
its longer leg connected by (i)
a 16 mm dia. rivets, (ii) a 6 mm
fillet weld.
Soln. : (i) Str en gth using 16
mm uia. riv~ts :
Gross diameter

d = 16 + 1.5
= 17.5 mm

1
90

l
t
Fig. 3.5
Design of Steel Structures [ 3-8 ] Design of Tension Member
Net sectional area of connected leg,

= s(90-11.s- ~)

= 548 mm2
Gross-sectional area of unconnected leg,

=448 mm2

3x548
=----
3x548+448

= 0.786

= 548 + 448 X 0.786


= 900.13 mm2
Strength = O'at x Anet

t.e. Pat = 150 x 900.13

= 135019N
= 135 kN
(ii) Strength using a 6 mm fill~t weld :
Net sectional area of connected leg,
~~ ign~~ ~~ ~~ ~-
pes of Stee l Stru ctur es -- -- -- _[ _~ [?J
3-9_
] __ ___!:D~e~s~iga:n:_:..of_T_e_n_si_o_n_M-:;.::.::ernb
~

= s(9o-!)
=6 88 mm 2
Gro ss-s ect ion al are a of unc onn ect ed
leg,

A2 = { b- ~)

= s(60-!)
=44 8 mm2

3x6 88
3x6 88+ 448

= 0.8216

= 688 + 448 X 0.8 216


= 1056.10 mm2
Strength = crat x Ane t
Pat = 150 x 1056.10

= 158415 N
= 158.415 kN
.... Ans.
No te : Co mp aris on of (i) anti-,(ii) abo
ve sho ws tha t tlct -ld ed con nec tion
stre ngt hav e more
h tha n the rive ted con nec tion . Howeve
r, it should be noted 't'tiat the qua lity of
workmanship
.in we ldin g wil l onl y pro vid e the abo
ve strength.
~=n-=-o~f-:S~te~e::l~S-:_tru~_c_n-:_1_r_e~s-::.-::.-::.-::.-:_-:_;-:_-:_-:_-:_-:_-:_-:_-:[-:.3~-~I_O~L]___________-_:-___. !_D~e~s:!.ci_·gn:_::_o::_f::_~T_=.:.e_n~s~io==n::~M;.e-::.m~_b:--e=r
~ .2 : Calculate the strength of a tie member composed of 2 ISA 150 x 75 x 8, when
they are :
(a) Placed back-to-back with their longer legs connected on the same side of
the gusset plate by 20 mm diameter rivets.
(b) Placed back-to-back with their longer legs connected on both sides of the
gusset plate by 20 mm diameter rivets.
Tacking rivets have been used.
Soln. : (a) Placed back-to-back on same side of gusset plate :

A1 = zi(t-d- ~J
= 2xs(1s0-21.5- ~)

= 1992 mm2

A2 = 2t( b- ~)

A2 = 2xs( 75- ~J
=1136mm 2

5x1992
= 5xl992+1136

= 0.8976

Anet =A1+A2•k
= 1992 + 1136 X 0.8976
= 3011.7 mm2
Strength = 150 x 3011.7

Pat = 451755 N = 451.755 kN .. .Ans.


D.:::es~ig~n~o::.f...:S:.:.:te=e=-1.::.St::.:ru::c.::tur::.:e::::,s_ _ _ _ _ __.L.:tl.!..1___ _~D~e~si£gn:,:...:.o_f_T_en_s_io_n;;,.,:M~emb
.::
~

T
150

+
l
150

Fig. 3.6
(b) Placed back-to-back on both sides of gusset :
C/s Area of single ISA 150 x 75 x 8

= 1748 mm2 (from steel table)


Anet =2 (Ag - deductio
. n for holes)

=2 (1748 - 21.5 X 8)
= 3152 mm2
Strength == 3152 x 150
=472800N
I.e. Pat = 472.8 kN. ... Ans.

T
150

f--75
---175--f

Fig. 3.7
Ex. 3.3 A tie consists of 2_ISA 75 x 75 x 8 mm placed back-to-back with a gusset plate
· 8 mm thickness between them. If 18 mm diameter rivets are used f~r the
connection, find the safe pulJ the tie can carry, if tack rivets are not provided.
Assume permissible axial tension stress = 150 MPa. What will be required pull if
.tacking rivets arc provided ?
. of Steel Structures
pcs1gn [ 3-12] Design of Tension Member
~ (a) When tacking rivets are not provided then both the angles act separately as a
. gJe angle :
Sil1

d = 18 + 1.5
= 19.5 mm
Strength of single angle

A1 = 8(75-19.5- ~)
= 412 mm 2

2
= 568 mm ISA75x75x8

T
75

·~ 75
-1
Fig. 3.8
3A 1
!r =
n 3A1 +A2

3x412
k = 3x412+568

k = 0.685

A net = 412 + 568 x 0.685

A net = 801.16 mm 2

Strength = 801.16 x 150


=120174N
r
Design of Steel Structures [ 3-13 ] Design of Tension Meinber
~
Strength of 2 ISA = 2 x 120.174
1.e. Pat = 240.35 kN .. ,Ans.
(b) When tacking rivets are provided :
Anet = 2 (A - deduction for holes)
g

= 2 (1138 - 19.5
Anet x 8)
Anet = 1964 mm2
Strength of 2 ISA = 1964 x 150

i.e. Pat = 294600 N

= 294.60 kN ... Ans.


Ex. 3.4 : One ISA 100 x 65 x 10 mm is to be connected to a gusset plate of
12 thickness
through a single rivet line. The rivet diameter is 18 mm. Show by calcu
lations that
it is advisible to connect the longer leg to the gusset plate. Take the
permissible
axial tensile stress = 150 MPa.
Soln. : (a) When longer let is connected :

d = dn +1.5=18+1.5=19.5 mm

A1 = 10(100 -10.5 -ln


A1 = 755 mm2

A2 = 10(6 s-
1
i)
A2 = 600 mm 2

k= 3x755
3x755+600

k ·= 0.79
Anet = 755 + 600 x 0.79
Anet = 1229 mm2
Strength = 1229 x 150

I.e. Pat = 184350 N or 184.35 kN

L
---~----~=-=-::__
Design of Steel Structures ______[L3i:-_!:
14Ll_]_ _ __JD~e~s~ig~n~o~f~T~e~n~si£on~M~e~m~b~er

T
100

l
~21+-
Fig. 3.9

When shorter leg is connected :

A1 = 405 mm 2

A2 = 950 mm 2

3x405
k=----
3x405+950

k = 0.561
Anet = 405 + 950 x 0.561

Anet = 938.14 mm 2
Strength = 938.14 x 150

Pat = 140721 N
I.e.
= 140.72 kN
Since strength of (a) > (b), it is advisible to connect longer leg to the gusset

plate.
Design of Steel Structures
[ 3-15 ] Desi n of Tension M
3.6 Design of Axially Loaded Tens elnoit
ion M em be r
The design of a tension member con
sists of selecting a section with suf
fi~
carry the factored design loads wi .
thout exceeding the design strength ~~
·• · of th e member.
design of a tension member is the mo 1
st simple and straight forwar d methodb ~e
eca use the stabili
of the tension member is of minor .
concern. However, the design proces . . ~
s 1s iterative. ·
Before designing a tension member,
1

I
the nature of loading and the types
which the member may be subjected of stresses to
are ascertained ; it may be su~jected
to either pure axial
tension or combined axial tension and
bending.
The computation of the required cro
ss-sectional area of a tension memb
enough. However, the proportionin er is sim-p\e
g and arrangement of the member and
its end connectioni
so that it is appropriate and econom
ical, may become quite involved. Fo I

r instance, it is often
difficult to arrange a connection wit
hout eccentricity, and stress concen
tration induced by the
connected parts can be extremely com
plicated. Although the designer has con
siderable freedom
in the selection of rolled shapes, the
resulting selection should have the
following propertiei:
1. Compactness.
2. Its dimensions should fit into
the overall dimensions of the structu
re.
3. Connections to as many parts
of the section as possible to minimize
shear laganct
stress concentration.
The tension member section once sel
ected should be ensured to have suffici
to prevent undesirable lateral deflec ent stiffnesi
tions and vibrations. For this, the sle
nderness ratio of the
member is checked and kept below
a eertain maximum specified value.
This also ensurei
some minimum specified compressi
on strength in the member to take car
e of stress reversals
during shipping and erection. and per
haps due to wind or earthquake.
An initial estimate or rn~ area is ma
de from the following conditions (1)
the larger one is taken as the initial and (2) ano
size estimate.
1. Gross section yielding :

Ymo
2. Net section fracture:
For plates and threaded rods
3-16] Desi n of Tension Member
of Steel
• ~.::
pes1gn
Struct
:.-- --- ures-- - --- --.1 ..... ::... .!~- ---~ ~~~ ~~~ :...: :::= ::~
limit, 'gross
Once the trial shape is selected, the section is checked for slenderness ratio
procedure for
section yielding, net section fracture, and block shear failure. The step-by-step
the design of tension member subjected to axial load is as follows :
1. The net area required An to carry the factored load T is obtain
ed by,

T T
An = 0 9f or by -f-
~ a u

Ym1 Ym1

t4 of
as appropriate. Where Tis the factor~d design load, fu is the ultimate streng
priate
the material, An is the net area of cross section, a= 0.6, 0. 7 or 0.8 as appro
and Ymt = 1.25.
ute the
2. The net area calculated thus is increased suitably ( 10% - 25%) to comp
tentative gross sectional area.
3. The trial gross area is also determined from its yield strength by

Ymo

where fy is the yield strength of the material and ymo = 1.1.


a cross-
4. From IS Handbook No. I, a suitable rolled section/built-up section providing
ed.
sectional area matching with the computed gross-sectional area is select
lated.
5. The number of bolts (or weld) required to make the connection is calcu
ded
These are arranged in a suitable pattern and the net area of the section provi
ed
is calculated. Thereafter, effective net area is determined if the section select
is not connected with all of its elements.
n.
6. The slenderness ratio of the member is checked as per the IS specificatio

Steps :
I. Calculate net sectional area required

Axial force
Anet required = Permissible tensile stress

2. ·. bl t' n having 10 to 40% larger than Anet required.


Try a suita e sec 10
r
Desig n of Ste el Str uct ure s
--, \

[ 3-17 ] De si n of Te nsi on Mernb


Fo llo wi ng_per cen tag e_s are rec er
om me nd ed for spe cif ic cas es
:
Co nn ec tio n ➔ Ri ve ted We lde d
An gle
,l.
Sin gle an gle
20 - 40 % 10 - 20 %
Do ub le an gle
20 - 30 % 5 - 10 %
3. Ca lcu lat e the Anet ava ila ble
in the trial section.
4. Th e tria l sec tio n wil l be sui tab
le if
'
An et ava ila ble ~ Anet req uir
ed (ne arl y wit hin 5% to 10%).
5. Ch eck sle nd ern ess rat io wh en
rev ers al of load may occ ur as
per IS : 800 - 1984 as
follows.
(a) In any ten sio n me mb er in wh ich
a reversal of dir ect str ess du e
to loa d other than
wi nd or ear thq uak e for ces occ
ur (SR ~ 180).
(b) A me mb er no rm all y act ing as
a tie in a roo f tru ss or bra dn g
sys tem by subject
to a po ssi ble rev ers al of str ess
res ult ing from the act ion of or
ear thq uak e forces
sho uld hav e S.R. ~ 350.
6. De sig n en d con nec tio ns

Ax ial force
No . of riv ets req uir ed = R'1vet va
1ue
Ar ran ge the riv ets as far as
pos~ible suc h tha t the cen tre
of gra vit y of the section
co inc ide s wi th tha t of gro up
of rivets.
Ex .3. 5 : De sig n an un equ al ang le sec tio
n to act as a tie me mb er 2 m
lon g in a roo f truss
if it is to car ry an axi al loa d of
150 kN. Th e con nec tio n can be
riv ete d wit h hand
dri ven riv ets or fill et we lde d.
Ta ke crnt = 150 MP a.
So lo. (A) De sig n us ing riveted conn
ection

_! _= 15 0x 10 00 =l 00 0 mm2.
I. Anet req uir ed = crat 15 0

2. Ch oo se an un eq ua l ang le sec
tio n hav ing a sec tio nal are a abo
ut 20 % mo re.
Ri st Trial sec tio n : ISA 80 x
50 x IO mm
A = 1202 mm 2
~-:-;~~:::::-=~_
pesign of Steel Structures
Use 20 mm diameter rivets _J[[13:!-1!8l]_ _ _J~~
Design of ~~~~ ~
Tension Member

d = 20 + 1.5
d=21.5mm

3. Ai = 10(80-21.5- I~)

A1 = 535 mm 2

A2 -_ 10 ( 50- 10)
2
A2 =450 mm 2

k= 3x535
3x535 +450

k = 0.781

T
80

l 50...j

Fig. 3.10
Anet == 535 + 450 x 0.781
2
Anet == 886.46 mm
Since Anc, ovailable > Anet required, section is unsafe.
Second trial section: Tiy JO% more area, hence choose an ISA 100 x 75 x 8mm having
2
A= 1336 mm

Ai = s(100-21. s- ~J
Ai == 596 mm2
~~~~~~ JS~~~~-
Qesign of Stee
true es --~---L~
_ 3-:21~9~]L.__ _ __:::D-=e.::.:si:5,:gn:,.__o_f_T_e_n_s_io...;n:...:M:.:.::::!
e~ - ----._r.

2
A 2 = 8 ( 75- ~)=568 mm

k = 3x596 =0.759
3x596+568
. . 2
Anet = 596 + 568 x 0.759 = 1027.06 mm

4. Since Anet available > Anet reqired, therefore, section is safe.

Strength of member, = 1027 .06 x 150 154.06 kN


pat 1000

5. Check for slenderness ratio

Effective length, le =2m = 2000 mm

rmin = 1.59 cm = 15.9 mm

- ~= 2000 =125.78l350
S.R. - rmin 15.9

which is alright.
(Note : A limit of S.R. ~ 350 is taken since the member is in a roof trust which carrie·s
forces from wind occasionally).
6. End connections
Strength of 20 mm diameter hand driven rivet

In single shearing = tvf x 7t x d2


4

Ps = 80x 7t x21.5 2
4
PS = 29044 N
In bearing = apf xd x t

Ph = 250 x 21.5 x 10

Pb = 53750 N
--ig-~ ~ ~ i- ~ ~be~r
~ em
2n~M
-:
3 = ~ ~ -- _ J ~ ~ Les
D n o f Tens io
_ e l_ S tr u _ c _ tu r_ e _ [ -20 J
s
pesig
::: - o_f_S_te
---~n~
Rivet value == 29.04 kN

== ~ -
.
( 6 nv
of rivets required - 5· 16 say ets)
No. 29_04

a pitch == 3 n _- 3 x 20 == 60 mm c/c
:. Provide 6 rivets at
mm
d edge distance == 2 dn == 2 x 20 == 40
an

I
-·-•- . ·-·
--l·-·-·-·-·-·-·-·-·-·-·-·-·
I
I

I ISA90 x 60 x 10 mm
\
-6 -2 0 mmf rivets

Fig. 3.11

d connection :
(Il) Design using welde

. 15 0x l0 00 1000mm 2
i ----
Anet reqmred = 150
l,

section which has 15 to 20% more area.


2. Try the
m having A = 1152 m m 2•
i.e. ISA 75 x 50 x 10 m
Size of fillet weld
(i) Minimum size
= 3 mm
3 3
toe section = t = x 10 =7.5 mm.
for rounde d 4 4
(ii) Maximum size 6
m m can be adop ted. Let us adopt a size of
een 3mm to 7.5
Hence the size in betw
ted to longer leg.
mm fiHet weld connec
connected Jeg
3. Net sectio r.~! area of

ss-sectional area of unconnected leg,


Gro

A t b- · (50- 210) =450 mm


= ( 2t) =10
2

2
Desi n of Steel Structures 3-21 ] Desi n of Tension M
'
ell1ber

3x700 ==0.824
k = 3x700+450

2
Anet = 700 + 450 x 0.824 == 1070.8 mm

4. Since Anet available > Anet required, the section is safe now.

Strength of member, - 1070.8 X 150 == 160.62 kN > 150 kN


Pat - 1000

5. Check for slenderness ratio

Effective length, le = 2000 mm

rmin = 10.6 mm (from steel table)

I 2000
S.R. = -er = l0 = 188.68 "J 350. O.K.
min •6

6. End connections :
Area of cross-section, A = 1152 mm2 (from steel table)
Load to be resisted, P = 150 kN

This load should act at the e.g. of the section, Cx = 26mm (from steel table)

Size of weld = 6 mm.


Effective length of weld required,

3
L = 150xl0
108x0.7x6 330.68 mm (say 335 mm)

The distribution of the weld should be such that the e.g. of weld coincides with the line
of action of load.
JJ~U- .---~~
·no:::,:f: S~te_.e_l_S_tru_c_ru_r_es_ _ _ _ _ _ _ [ 3-22 ]
~ Design of Tension~~~~
~!.£Q~ Member

I
I
p I

·-·-·-·-·-·-·-·-·-·-·-·~------·-·-·---.p· 1
1i~rr_rr_fiirimnnrrriir==::::::;;::J:' ==d· · -·-2e'mm·-·-·-·-·-·-·-·-·-·
T
Fig. 3.12

From Fig. 3.12, total effective length of weld


X1 + X2 + 75 = 335
x 1 +x 2 =260 mm
Force in weld per mm length

= Pq xix t
= 108 X 1 X 0.7 X 6
= 453.7 N/mm
Taking moment of forces about bottom edge of member
75
453.7xx 1 x75+453.7 x75x = 150xl03 x26
2
X1 + 37.5 = 113.61
x1 = 77.11 say 80 mm •

x 2 = 260 - 80 = 180 mm
Ex.3.6 : Design a suitable single angle section as a tie member to carry a tensile force of
100 kN to be connected by 16 mm diameter rivets to 10 mm thick gusset pl~te.
Refer Table 3 .1
Table 3.1 : Available R.S. Section

Section Sectional Area

817 mm 2
ISA 65 x 45 x 8 mm
938 mm 2
ISA 75 x 50 x 8 mm
1100 mm2
ISA 75 x 50 x 10 mm
r
Design of Steel Structures [ 3-23
Desi n of Tension '
. 10 2
Soln. : 1. Anet reqmred = 0 X 1000 == 667 mm
150
First trial section :
2. Try section ISA 65 x 45 x 8
mm having .
A = 817 mm2, d = 16 + 1·5 == 17.5 mm

A1 = 8 65 -1 7. 5- 28J =348 .mm 2


(
A2 = s(45- ~)=328mm 2

k= 3x348 ==0.761
3x 34 8+ 32 8

Anet = 348 + 328 X 0.761

Anet = 597 .6 mm2


Since Anet available > Anet
required, therefore section is
unsafe~
Second trial section :
Try other section ISA 75 x 50 x 8 mm
having A = 938 nn 12

A1 = 8( 75 -1 7. 5- ~)
A1 = 428 mm 2

A2 = so(so- ~)
A2 =368 mm2

k == 3x 42 8 _
3 X 428 + 368 -0 .7 77

. 3. Anet = 428 + 368 x 0.777 == 714 mm2


.!_f~T~e~n~si~o~n_:M~em~b~er
3-::2~4J]L__ __D~es~iB;gn~o'
~~ig:::..,n_o_f_S_te_e_l_S_tru_c_tu_r_e_s_ _ _ _ _ _ _1[ ~

4. Since Anet available > Anet required, therefore section is ·safe.

Strength of member, Pat = 714 x 150 = 107100 N = 10.710 kN

E.µd conn ectio ns : Strength of 16 mm diameter hand drive


n rivets.
5.

In single shear, PS = s·ox 1t x (17 .5) 2 =19242 N


4

In bearing, P0 = 250 x 17.5 x 8 = 35000 N

Rivet value, Rv = 19.24 kN

P 100
No. of rivets requird =-=_ ,___ =5.1 97 (say 6 rivets)
RV 19.24

Pitch = 3 dn = 3 x 16 = 48 say 50 mm c/c.


Edge distance = 2 dn = 2 x 16 = 32 say 35 mm.
two unequal
Ex.3 .7 : Design a tension member subjected to an axial pull of200 kN by using
angles with suitable tacking rivets.
same side of
(a) Placed back-to-back with their longer legs connected on the
the gusset plate by 16 mm ~ pdf rivets.
of th~
(b) Placed back-to-back with their longer legs connected on both sides
gusset plate by 16 mm ~ pdf rivets.
:
Soln. : (a) Desi gn with angle s on same side of gusse t

3
200x l0 = 1333 mm2
1. Anet req . = 150

2. Try 2 ISA sections having about 20% more cross-sectional area.


g area of each angle
i.e. 1.2 x 1333 = 1600 mm 2. Thus, try 2 ISA 90 x 60 x 6 mm havin
= 865 mm2.
3. Net sectional area
d =16 + 1.5 = 17.5 mm

I,
~
~~,£_§~~~~,__ _ _ _ __J~~
J3 -25 ]- -:~- ~D
~e~s~
ig~n_o_f_T_e_n_si_o~nMe
Design of Steel Structur t:s ~,
)
A, = 2x6(9o - t7.5 - ~
2
Ai == 834 mm

A2 = 2x6( 60- ~)

A2 = 684 mm 2

5A1 - 5 X 834 = 0.859


k = 5A +A 2 - 5x834 +684 . I
1

Anet avail. == AI + A2 k

= 834 + 684 x 0.859 = 1421 mm2

4. As Anetavai l. > Anet req .' section is safe

Strength, Pat = Anet x crat


= 1421 x 150 = 213150 N = 213.15 kN :. 0.K.
5. Design of end connections :
Strength of one pdf rivet.

In single shear Ps = 7t d2 Xtvf = 7t x17.5 2 x90


4 4
PS = 21647 N = 21.64 kN

In bearing, Pb = dxtxcrpf = 17.5 x 6 x 270

Pb = 28350 N = 28.35 kN
·· Rivet value, RV = 21.64 kN

P 200
No. of rivets Rv = 1. .= 9.24 say 10
2 64
11111111"--

~ of Steel Structures [ 3_26 ] Design of Tension Member

Minimum pitch P = 2.5 dn = 2.5 x 6 = 40 say 50 mm


Edge distance, e = 2 dn = 2 x 16 = 32 say 40 mm

T
90

t
90
21SA 90 >< 60 >< 6 mm

l so-{

Fig. 3.13
L 10 -16 rrwn • hand driven r1ve11

(b) Design with longer legs of angles connected on both sides of gusset :

200x10 3 2
1. Anetreq. = lSO =1333 mm
2
2. Try 2 ISA sections having about 20% more area. i.e. 1.2 x 1333 = 1600 mm •
2
Thus, try 2 ISA 90 x 60 x 6 mm having area of each angle = 865 mm •
3. Net sectional area
d = 16 + 1.5
= 17.5 mm
Anetavail. = 2 (Ag - d.t)
= 2 (865 - 17 .5 X 6)
= 1520 mm 2
4. As Anetavail. = Anetreq. section is safe

Strengtb, Pat = Anet x O' at

= 1520 X }50
= 228000 N
= 228 kN > 200 kN . . O.K.

5. Design of end connections :


Use 8 mm thick gusset plate
J
of Tension
Design of Steel Structures [ 3-27 ]
Strength of one pdf rivet
. 1t 2
In double shear p == 2 X -d X 1 vf
ss 4
p == 2x 7t x17.52 x90
ss 4
pss == 43295 N

== 43.29 kN
In bearing, Pb == d x t x crpf
17.5 X 8 X 270

Pb == 37800 N
= 37.80 kN
Rivet value, Rv = 37.80 kN
. P 200
No. of rivets = - = - - = 5·29. .
6 No.
RV 37.80 say
Min. pitch p = 2.5 dn = 2.5 x 16
= 40 say 50 mm.
Edge distance, e = 2 dn = 2.5 x
16 = 32 say 40 mm.

I90
I
I
I
I

1 •• I .

·~ t+- · 6 -16 mm ♦ pdf rivets


8mm 2 ISA 50 x 60 x 6 mm
Gusset 8 mm thick

Fig. 3.14
Ex.3.8 : Desing a T-section to act as a ten
sion member carrying an axial ten
sion of 220
kN.

220x1000 -1466.7 mm2


Solo~ : 1. Anetreq. = 150
pesign of Steel Structures
;.----
. · ISHT 75 @ - ~
Choosmg [ 3•28 ] . of Tens10n
Des1gn . Member
2 15 .3 kgf/m, having t. 2
18 mm diameter hand d . . sec tonal area = 1950 mm connected by
riven rivets.

d === 18 + 1.5 = 19.5 mm

A1 === tf(bf - 2d)

A1 === 9 (150 - 2 x 19.5) = 999 mm2

A2 = tw (b - tr)

A2 -- 8.4 (75-9) = 554.4 mm 2

k= 5x999 _
5 X 999 + 554.4 - 0. 9

3. Since Anet = 999 + 554.4 x 0·9 = 1498 mm2 > 1466.67 mm2, therefore section is safe.

T
I
Fig. 3.15

4. End connections : Strength of 18 mm diameter hand driven rivets.


2
In shearing; :=>s =8Ox ;x19.5 =23892 N = 23.89 kN

In bearing, Pb =25Oxl9.5x9 = 43875 N = 43.88 kN

Rivet value, Rv =23 .89 kN

Number of rivets required = ~~i =9 9.21


(say 10 rivets)
r
Desi n of Tension Me
[ 3-29 ] 55 lllbet
Design of Steel Structures
. = 3 x l 8 = 54 say mm and
Provide 10 rivets in pairs at a pitch 3 dn
. _ 6 ay 40 mm.
edge distance = 2 dn = 2 x 18 - 3 s

~3.2_7___!:Lu~g~A~n~g~le~ - - - - - - - - - - - - - - - - . . . . _ _ _ _
Gusset plate.
· Lug angle

. 0 0 0

1---- Angle section

Fig. 3.16
The lug angle is a short piece qf an angle section used at a joint to conn~ct the outside
leg of a member, thereby reducing the length of the joint. Gusset plate material is saved by
using a lug angle ·but extra material is required for lug angle and their connections. Lug angles
are also not very efficient in transmitting loads and a certain eccentricity is caused between
the load and the centre of gravity of the river group. The use of lug angles is·therefore avoided
in general. A lug angle is provided at the beginnin~ of a joint so that it can be effective in
sharing the load. The .following specifications are followed for the design of lug angles
as per IS : 800 - 1984 clause 8.8.
1. Lug angles connecting a channel shaped member should as far as possible, be disposed
symmetrically with respect to the sec~ion of the member.
2. In the case of angle members, the lug angles and their connections to the gusset or any
other supporting member should be capable of developing strength not less than 20%
in excess of the force in the outstanding leg of the angle and the attachment of the lug
angle member should be capable of developing 40% in excess of that force.
3. In the case of channel sections, the lug angles and their connection to the gusset or any
other sµpporting member should be capable of developing a strength of not less than
10% in excess of the force not accounted for by the direct connecti.on of the member'
,
and the attachment of the lug angles to the member should be capable of developing
a 20% in excess of that force.
4. In no case should fewer than two bolts or rivets be us~d 1or
c . angle
attachmg the 1ug
to the gusset or another supporting member.
. of Steel Structures Design of Tension Member
~ [ 3-30 ]
The effective con nec tion of th e Iug angle should as far as possible terminate at the end
5.
of the mem ber con nect ed and. th e 1castenmg

of the lug angle to the member should
of th d' •
Preferably star t in advance e rrect connection of the member to _the guss
et or
other supp orti ng member.

member the whole area of the member
6. Where lug ang les are used to connect an angle
should be take n as effective, i.e.

Anet = Gross area - Deduction for holes.

3.8 Tension Splices


sections when a join t is to be provided,
, Splices in tens ion mem ber are used to join two
re it is cut, where a splice occurs in an
i.e. these repl ace the members at the join t whe
abstracted from the section through the
angle, chan nel, Tee or join t section, the force is
sferred to the splice cover plate/angle
connections on one side of the join t and is tran
section.
across the join t and is transferred to the
The force is then carried through these covers
n.
other port ion of the section through the connectio
riveted connection.
• Splice con nec tion for tension members is the
for the tensile load to be transmitted
, The splice sect ion as well as the rivets are designed
by the main tens ion member.
should be designed for the maximum
• There is alw ays a question whether the splice
the member.
calculated force or for the actual strength of

(a)
~(b) (c)

-
II t
Packing

:ti l I ! g II

111 ~

- -

(d) Fig. 3.17


Design of Steel Structures [ 3-31 ] Desig n of Tensi on Member
~
• As per I.S . specification, the splice covers and its connections shoul d be
designed to
develop the net tensil e strength of the main member.
• The variou s types of splices that can be provided are shown in Fig. 3 -17
(a, b, c).
• If the sectia ns are not of the same thickness, packings are introd uced as
shown in
Fig. 3 .17 ( d) and the extra rivets are calculated by providing additi onal 2.5%
of total
rivets for each 2 mm thickness, if packing thickness is more than 6 mm.
• Normally in the design of a tension splice, the effect of eccen tricity is neglec
ted but as
far as possible it should be avoided. Fig. 3 .17 (a, e) show an angle sectio n
spliced_on
one leg by the plate.
• Such an arrangement causes eccentricity and, thus, moments are create d. To
overcome
this both the legs of the angle should be spliced, as shown i Fig. 3 .17 (b ).
Fig. 3.17
(c, d) show the splicing of plates.

I IMPO RTAN T POIN TS I


• A memb er carrying direct tension is called as a tie or tension member.
• Tension members are found in roof trusses, towers, bridges and bracin g system
s.
• The strength of tension members or load carried by it is given by

IPat = crat x Anet I


where, Pat = Axial Tensile load/strength {N)
crat = Permissible stresses in axial tension (N/mm2) generally
150 MPa mild steel.
Anet = Net sectional area (mm2)
• NET Sectional Area
(i) For single angle connected by one leg only
. 3A
1
Reduction factor, k = A + A
3 1 2

(ii) For pair of angles placed back to back connected by one leg of each angle to
the
same side of a gusset and tack riveted outstanding legs.
Design of Steel Structures r 3-32 I Design of Tension Member

For both the above cases

!Anet = A1 + A2 kl
(iii) For double angles or tees placed to back to back and connected to both sides of
gusset and tack riveted in between.

• Lug Angle is a short piece of an angle section used at a joint to connect the outstanding
leg of a member so as to reduce length of joint
• Tension splices are used to join two sections when a joint is to be provided.

I PRACTICE QUESTIONS I
'
1. State allowable tensile stress in steel as per IS 800.
2. Enlist four types of section used as a tension member.
3. For unequal angle tension member state with reason whether longer leg connected or
shorter leg connected be economical.
4. state function of tacking rivets.
5. Calculate net area of a flat I 00 x 10 mm having hole 20 mm diameter drilled through it.
6. Calculate the strength of ISA 40 x 25 x 6 mm thick when used as tension member with
its longer leg connected by 14 m~ diameter rivets.
7. Calculate the net effective area of o~e ISA (90 x 90 x 08) mm used as a tie member·
connected to a gusset plate of thickness 10 mm, using 16 mm diameter rivets.
8. In a roof truss, a diagonal consists of an ISA 60 x 60 x 8 mm and it is connected to
a gusset plate by one leg only by 18 mm diameter rivets in one chain line along the
length of the member. Determine the tensile strength of the member.
9. A double angle tie ISA 150 x 75 x 10 mm (short legs back-to-back) of a roof truss is
connected to the same side of a gusset with rivets 18 mm in diameter, such that each
angle is reduced in section by one rivet hole only. Determine the tensile strength of the
member. Tack rivets have been provided at suitable spacing.
Design of Tension Memb
'
Design of Steel Structures [ 3-33 ] ~
.
1O. In Ex. 9 above 1f the angles are connecte d to eac h side of a gusset, ·determine the
tensile strength of the member.
·
11. A pair•
of angles 125 x 75 x 8 mm back-to-bac'> an c
1,, d onnec ted with their smaller Iegs
·
on the same side of a gusset plate 10 mm th"1ckbY n·vets 20 mm in diameter· Lo
nger '
legs have been suitably tack riveted. Determine the tensile strength of th e membe
r.
12. Double angles 80 x 80 x 12 mm back-to-back and connected on both sides of a gusset
plate 12 mm thick by. rivets 20 mm in diameter. Outstanding legs have been stitched
together suitably. Determine the tensile strength of the member.
13. A tension member consisting of 4 ISA, 100 x 100 x 10 mm is connected to the gusset
plate by 18 mm diameter field rivets. Find the net effective area of the section in
each
of the following cases:
(i) Angles are tack riveted along all connecting and outstanding legs.
(ii) . Angles are not tack riveted.
14. Find the effective net area of a tension member in the following cases :
(i) 2 ISA 75 x 7510 mm, placed back-to-back and connected to the same side of
x
gusset plate of 12 mm thickness. 20 mm diameter rivets are used for connection,
to tack rivets are not used.
(ii) Arrangement .of 2 ISA in (i) above, but tack rivets are used.
15. Find the net effective area of a tie member consisting of 2 ISA (90 x 90 x 108) mm
connected to one side of a gusset plate of thickness 10 mm using 16 mm diamete
r
rivets. Tack rivets were used.
.
16. A tension member has 2 ISA 80 x 50 x 6 mm connected to the longer side and on the
same side of the gusset plate. Calculate the .load carrying capacity if the angles are:
(i)
Tack riveted, and (ii) not tack riveted. (Use 18 mm cp rivets)
17. A tie member consists of two ISA 100 x 100 x 10 mm connected back-to-back
at ·same
face of the gusset plate using 16 mm diameter rivets. Calculate the net area if:
(i) Tack rivets are provided at suitable spacing.
(iQ Tack rivets are not provided.
18. A double angle tie ISA 125 x 95 x 10 mm (short leg ba~k-to-back) of a roof truss
is
connected to the same side of a gusset with rivets 20 mm in diameter: such that each
angle is reduced in section by one rivet hole only. Determine the tensile' strength of.the
member. Taking rivets have been provided at suitable spacing. Take crat = 1_50 MJ>a.
19. A tension member consists of two ISA 100 x 100 x 8 mm connected back-to-back
opposite faces of 10 mm thick gusset plate. Using 20 mm diameter rivets, calculate
its
load carrying capacity if :
Design of Steel Structures
- (i) Tacking rivets are provided.
[ 3-34 ] Design of Tension Member

(ii) Tacking rivets are not provided. Take cr = 150 MPa.


81
of 150 kN.
20. An unequal angle section 125 x 75 mm is required to cany a tensile load
by 18 mm
The angle section is connected to the gusset plate through the longer leg
carrying
diameter rivet Suggest suitable thickness for the angle and check for load
capacity. cra, =150 MPa.
roof truss if
21. Design an unequal angle section to act as a tie member 1.56 m long in a
fillet welds.
-it is to carry an axial load of 120 kN. (i) Use hand-driven rivets at joints, (ii)
carry a pull
22. Design a single equal angle section for a tension member of a roof truss to
to action of
of 100 kN • The member is subjected to possible-reversal of stress due
3.50 m. ·
wind. The length of the member from centre ·to centre of intersection is
cted by
.23. Design a tension member consisting of a pair of angles (back-to-back) and conne
of 250 kN.
short legs to the same side of gusset plate. The member is to carry a pull
~esign the
24. Design a tie member of a roof truss carrying a tensile force of 90 kN. Also
riveted connection. Use I.S. specifications.
of 165 kN.
25. Design a double angle tie member for a light roof truss carrying a tension
heat sketch
Also design its end connection by using 18 mm diameter rivets. Draw a
showing the details of the joint.
angles connected
26. Design a tension member to carry an axial tension of230 kN. Use two
ications.
on the same side of the gusset plate by 18 mm diameter rivets. Use I.S. specif
unequal
27. Design a tension member subjected to an axial pull of 200 kN by using two
angles:
(a) Placed back-to-back with their_longer legs connected on the same side of.the
gusset plate by 16 mm rivets.
the gusset
(b) Placed back-to-back with their longer legs connected on both sides of
plate by 16 mm rivets .
T section is
28. Design a T section to carry a tensile force of 180 kN. If the flange of the
= 250 MPa.
connected with the gusset by rivets, find the number .of rivets required if fy
cted with
29. Design a single angle tension member to carry a pull of 220 kN. It is conne
in the weld
gusset plate using fillet weld. Allowable tensile stress is 140 MPa and that
is 108 MPa.
plate using
30. Design a tension member using single angle section connected with gusset
MPa.
fillet weld. It carries a force 190 kN. Take 0-81 =150 MPa and Pg = 108
□□□
Chapter
Des ign of Com pres sion Mem ber J
Introduction

A compression member is a structural member which is straight and subjected to two


equal and opposite compressive forces applied at its ends. An ideal compress ion member is
one which is perfectly straight, has no crookedness, no imperfections; and the loads are applied
uniformly across it, with'. tqe centre of gravity of loads coincidin g with centre of gravity of the
member. Such a compression member will be a truly axially loaded member. However, the
situations as stated above are impossible to achieve and a perfectly axially loaded compression
member will ever be encountered in practice.

Different terms are used to designate a compression member dependin g upon its position
in structures. Column, stanchion or post is a vertical compression member supportin g floors or
girders in a building. These compression members are subjected to heavy loads. Strut is a
compression member used in the roof truss and bracing. It is small span and lightly loaded
compression member. A strut may be continuous or discontinuous. A continuou s strut is a
compression member which is continuous over a number of joints, such as a top chord member
of a truss bridge girder, principal rafter of a roof truss, etc. A discontinuous strut is a compression
member which extends between two adjacent joints only, e.g., vertical or inclined compression
members in a roof truss. The principal rafter is a top chord h~ember in a roof truss and boom
is the principal compression member in a crane.

Stability plays an important rple in the design of compression members. Ordinary structural
analysis is based on the condition of stable equilibrium between internal and external forces,
and a linear relationship is assumed to exist between stress and strain. However when buckling
is involved, it is necessary to investigate the potentially unstable equilibrium between the external
and internal responses that are further complicated by the complex stress-strain relationship of
the material extending from elastic to inelastic range. The term unstable used here pertains to
a condition in which the slightest increment of deflection results in a further increase, which
pesignof Steel Structures
---- [ 4-2] Design of Compression Member
ay lead to collapse of structure Co . ·
m . · mpress1on members are actually more critical than tension
members, because if even slightly b t . ·
. . en compression member is placed in structure it may have
.
ignificant bendmg moment (equal tO the co lumn load times
s the initial lateral deflection) whereas
a tension member will try to straighten out.

Columns are sometimes classed as long, short, or intermediate. It should be taken note
of that th e terms long, intermediate and short columns are only relative. They are defined by
the interpretation of their slenderness ratio. Excessive compression of long columns may cause
yielding or buckling. It can fail due to yielding if it is abso-lutely straight, has perfectly
homogeneous material, concentric loads and no initial residual stresses. These are ideal conditions
which may never exist in an actual struc-ture. As compressive load on a column is increased,
it eventually causes some eccentricity. This in tum sets up some bending moment, causing the
column to deflect or buckle slightly. This deflection increases the eccentricity and thus the
bending moment. This may progress to a point where the bending moment is increasing at a
rate greater than the increase in load, and the column fails soon by buckling.
In general, long columns fail by elastic buckling, intermediate columns by inelastic buckling
(yielding and buckling), here EI changes continuously, and very short columns usually fail by
crushing or yielding. At the point of failure, the stress in a long column will not exceed the
proportional limit and it may be much lower than this limit for a very slender column. Failure
of the intermed iate column occurs after the extreme fibers have reached the yield point and
the others remain elastic. A very short column is not really a column as such but is considered
to be a block without buckling. A short column under applied axial force is subjected to
compressive strain; the column shortens in the direction of applied force. When the loa~ is
increased gradually, the shortening of the column continues until the column squashes. Similar
axial shortening is observed for long columns also but only in the initial stages of incremental
loading. Thereafter, as the load is increased further, it becomes unstable; the column buckles.
This chapter will discuss the design of individual compression members in order to illustrate
the detailed procedur e involved in selecting appropriate proportions of such members.

4.1 Definition
.iected to axial compression is called a compression member.
• A structura l mem b er s Ub 'J

truss carrying compression are popularly known as struts .


• The compone nts o f a .
. b · a building carrying direct compress ion are called columns,
• The vertical mem ers 10
stanchions or posts.
• '
Design of Steel Structures [ 4_3 ] Desi n of Com ression ~1etnb
et
4.2 Types of Sect ions ~
· ·
• The different types of sections used as compression members are shown in Fi g, 4.J
'Y
I

!Y ,Y
I i

lL
i
. I
X·- · ·-·-X X·- ·· · , i
! ·.
·- ·-·-X
.I I
.
I
!Y !y
iY
(a) (b) (c)
!Y !Y !Y
-------+-
apm , -----
· ·-·
I
I
I

X ....... -. ·- · Ii . ·-· • . ·-·-··X

___ ,
-------..------
i
i
i

ly ly jy

(d) (e) (f)
!Y ..!Y

~~~r1-y~~-
I
1-
!
I
,.
. ! ,~-· . .
I .
i i
; ! i j . I .
.X•-··· ' ·-,- ·' ·- ·- ·-X
X·- ·- · '1··1-
! ;; ·-·- ·-X X - .:•-·- ·- · -·· -~ ·- ·- ·- .•-·- ·- X I
' i . I I
i .i i . .•
! i. i .
! ! i '.
I Uliiilliii itiilL-,,-
iY j y
(g) (h) (I)
Fig. 4.1 : Various Forms of Compression Members

4.3 Effective Leng th


The form of curve into which a compression meml r tends to deflect depen
ds upon
the mode of end fixtures. In each case there is a portion of the length of the
compres-sion ·
member which bends as if this part had been a pin-jointed end. The end points
of this portion
of the compression member are the points of contraflexure. The effect ive
length, /, of a
compression member is the distance between these points. Therefore, it shoul
d be derived
from the actual length and end conditions. The end conditions are accounted
for through the
use of effective length factors K, which when multiplied by the actual length L give
the effecti'1e
length. The magnitude of the effective length factor depends on rotational restra
ints supplied
•at the ends of compression member and upon the ·resistarn;e to lateral move
ment provided,

pesign of Steel ~tructures 4-4 Desi n of Com ression Member
n member a precise
· ce the effect ive length is based on the end conditions of the compressio
sin assumption may 1 d
determination of th e effective length is very difficult and any arbitrary
of columns and struts
to serious errors . Tables 1 and 2 provide the effective length (KL)
length of a particu-Jar
respec tively, for various end conditions. The smaller the effective
the greater is its load-
compression member, the smaller is the danger of lateral buckling, and
carrying capac ity.
ined for com-
The concept of effective length in the above paragraph has been expla
columns never exist in
pression memb ers supported only at their ends. However, in practice
int (e.g., by bracing) to
isolation. If there is a possibility of ·providing additional lateral restra
length of the members
strengthen the weaker axis of the compression member the effective
can be further reduced and consequen tly the buckling load increased.
If a compression mem-
the effective length will
ber is supported differently with respect to each of its principal axes,
braced columns in a
be different for the two directions; poles carrying electric wires and
in Fig. 1 the column is
buildi ng are but a few examples. For the pin-ended column as shown
brace has been provi ded
supported differently with respect to each of its princi pal axes as a
weak axis of the cross
at mid-height of the column preventing translation perpendicular to the
atically in Fig. 1, if the
section. Consequently the weak axis is strengthened. As shown schem
be L, while for buckling
member is to buckle about the major axis, the effective length would
about the minor axis the effective length would be L/2.
~

I I
I
I
I

'
I
U2
I

U2 I '
'
'
L I
L
I

U2 '
I
U2 Column
I
I
I
I I
I web

" KL= L "


KL=U2
(a) Minor axis buckling (b) Major axis buckling

Fig. Column supported differently with respect to its principal axes

It is defined as that of column for which it acts as if both the ends


are hinged. At these

ce between two
points, the flexure changes its sign or in other words it is the distan
points of zero moments.
end conditions .
• The effecitve length / is derived from the actual length L for different
According to IS : 800 - 1984, it is taken as given in Table 4. 1
D~'si •n of Steel Structures De si m of Com ression
,
4-5 Me \
To bi e
4.1 : Effective Length of co ll1bet
mpreHlon Men1ber• of eo
na ta nt Dimension, '
. (IS 80 0 § 4.2 .2)
Degree, or end reatralnt
of compression Recommended va lu e
member SVrnbol
of effectlv:::.
• .:.:•e=n::.!g:..:_th -t-- - - r - -
(a) Effectively held In
position and
res trained against ro tat
ion at both
en ds.
0.6 5 L

(b ) Effec tiv ely held ln


position at • both
en ds · restrained again
st ro tat ion at
'>n e en d.
0,8 0 L

(c) Effectblely he ld tn· po


sition at both
ends, bu t not· restraine
d against
ro tat ion .
1.0 0 L

{d) Ef fec tlv eiy hel~ ·In position an d


re str ain ed ag ain st ro tat
ion
end, and af th e oth er end resat one
trarned
ag ain st ro tat ion bu t 1.2 0 L
no t _held in·
. position.

·_(e) Effectively hel~


lo position t,nd
res tra ine d ag ain st rotat
end~ an d at ion ·.at
one
th e oth er end , partially
restrained agal_n st ·rotat 1.5 0 L
ion bu t . no t
held In po sit ion .

: ,_(f) Effectively he ld · In
and bu t
position at •·
no t restrained aga'1r1s
one
ro tat ion , and at · the · t
oth er · .end
restrained ag ain st .-ptat
lon bu t · not :z.oo L
held In. position. .

:,(9) Effeqlvely held In position · and


restrained rotation at
on e end bu t
no t held In position or
res
ag ain st rotation at -ttie ot he trained 2.0 0 L I
r end. I
I
I

Note For effective length of stepped


columns, Appendices D-2 and
D-3 of IS: 800
may be referred to.

The K values in Table 8.1 are


satisfactory for use in designing
for the co lumns in continuous fra isolated columns, but
mes, they are satisfactory for makin
g p~eliminary or approximate
[4 . ,.
f steel Structures -6 ] Design of Co mp ress10n Member
~ 1
fra me s ma y be br d frames sup por t the latera
desig11 5
0 nly. These•
, . ace or unbraced. Unbraced
st tffness of its members and the rotational rigidity of the joints between the
oads
J bec aus e of
ven t late ral buc klin g. Suc h colum ns are restrained at their ends by their
{rarne members to pre
s bea ms , and the bea ms the ms elv es are connected to other columns and
connections to variou
0th e nd s and thu s also rest rain ed. The se connections can appreciably affect
beams at their er
K values in Table 1 are not sufficient
for
As a res ult, for mo st situ atio ns the
the K values.
Co nsi der the unb rac ed rigi d fram e of Fig. 2. The columns of the frame are not
final designs.
t me mb ers but par t of a con tinu ous stru cture. Except for the columns in the lower
inde-penden
the col um ns at the bas e are not rest rain ed in rotation these are restrained at both
storey where
sway
by the ir con nec tion s to bea ms and oth er columns. Since the frame is unbraced, side
the ends
umns
sibl e. The low er sto rey col um ns can thu s be approximated by K = 2 while other col
is pos
ted by K = 1.2 . A mo re ratio-n al pro ced ure, however, will account for
can be best approxima
members.
degree of restraint provided by connecting

Column AB:
End Translation Rotation
A Free Restrained
8 Restrained Restrained
m
C

D
· Column CD: ·
End . Translation Rotation
C Free . Restrained
D Restrain ed Free
D.

(b) Schematic n,presentation


(a) Unbraced plane frame

Fig.
A multi-storey unbraced frame

braced as shown in Fig. 8.3. The addition~}


Often frames of multi-storey structures are
ral loads is der ived from diagon al bra cing or shear walls or from adjoining
resistance to late some
es. Co lum ns of a bra ced rigi d fram e are prevented from side sway and have
structur
. trat'nt at the ir end s. K value for. such columns varies between 0.5 and
degree of .rotat1ona1 res
Design of Steel Structures Desi n of Com ression .
[ 4-7 ]
1.0. A value of 1.0 is _therefore always conservative unless an
analysis is made. The frame must resist not only the tendency
. to sway under the action of lateral loads but also the ten-dency
to buckle, or become unstable, under the action of vertical loads.
Bracing to stabilize a structure under vertical loading is called
stability bracing.
A frame in which lateral sway is prevented is called a
(a) Diagon al bracing
non-sway frame. Further, frames that sway to a small amount
and have a negligible P-A effect are also called non-sway
frames. Therefore, to define the term non-sway precisely the
criteria used is lateral stiffness irre-spective of whether it is
braced or not. Lateral stiffness can be provided either by making
the joints rigid, or bracing the frames, or shear walls. The inter- N

(b) Shear walls


storey drift- the difference in deflection of top and bottom ends
Fig. Braced frames
of a column in that sto-re y-qua ntifie s the lateral stiffness of
the frame.
As already mentioned isolated columns are rare and are norma lly parts of frame-w
ork.
A fairly accurate estimation of their K values requires the application ofmet h-ods
of indetenninate
structural analysis. These procedures are practically not fea-sible and simpl e
models are used
for estimating the K values. As per IS: 800, in the absence of a more exact'
analysis, the
effective length of columns in framed structures may be obtained by multip
lying the actual
length of the column between the centers of '.aterally supporting memb ers
(beams) with the
effective length factor K, calcu-lated by using the equations given below, pro~id
ed the connection
between beam and column is rigid type. These equations are based on Wood
's curve.
Non-s way Frames (Braced Frames) : The effective length factor K -of
column in
non-sway ·frames is given by

K = [l+0.145(f31 +P2)-0.265J31 P2]


2 - 0.364 CJ31 + J32 ) - 0.247 p p ... (l)
1 2
Sway Fram es (Moment Resisting Frames) : The ·effective length factor K
of coluilln
in sway frames is given by

K = [1-0.2(1\ +J3i)-0.1213ill2]0.5
i -o.s CJ31 + P2)-0 .6 P1 P2 ... (2)
w h e re A A
t-'l' t-12
(coefficients at top and bottom ends of colum ) . .
n are given by
pesiga:,:n_o_f_S_te_e_l_S_tru_ct_u_re_s_ _ _ _-:::--4-8-l~ l_- -~~ ~~~~~~ ~~~~r
;:.--- [ ] Design of Compr ession Membe
f3i :::: LKC
tKC +tKb .
Kc, Kb = effective flexural stiffn
. . ess of the. columns and beams respectively meet-ing
at the Jomt at the ends of h
t e columns and rigidly connected at the joints,
and these are -calculated by

Kc = C(I/L); Kb === y(I/L) (Beams are assumed to be fixed at far end; y = 1)


I = momen t of inertia of the mem b.er about an axis .
. perpendicular to the plan
of the frame

L = length of the member equal to centre to centre distance of the intersect-ing


membe r

C = connection factor
Notes :

1. For column s fixed at the base, rKb = oo and hence (3 2 = O.

2. For column s with a hinge at the base, rKb = 0 and hence (3 2 = 1.

3. For very slende r column connected to girders of large cross sections, the girders
will prevent the rotation of the column resulting in approximately fixed end condition;
K wil I be relatively a small value.
, .

4. Stiff column connected to flexible beams can more freely rotate and approach the
pinned end condition; K will be relatively a large value.
Table : Connection factor
Connection Factor C
Far End Condition
Braced Frame Unbraced Frame

1.s (i-n) 1.5 (1- ii)


Pinned
1.0 (1-ii) 1.0 (1- o.2ii)
Rigidly con _:·• cted to column
2.0 (1- 0.4 ii) 0;67 (1- 0.4 ii)
Fixed

p P = applied load
where n -
Pe
L = unbraced length of column
_E I lastic buckling load = 1t2EIII.:
Pe - u ere

You might also like